T10 Question

100
Certified Accounting Technician Examination Advanced Level Time allowed Reading and planning: 15 minutes Writing: 3 hours This paper is divided into two sections: Section A – ALL TEN questions are compulsory and MUST be attempted Section B – ALL FOUR questions are compulsory and MUST be attempted Do NOT open this paper until instructed by the supervisor. During reading and planning time only the question paper may be annotated. You must NOT write in your answer booklet until instructed by the supervisor. This question paper must not be removed from the examination hall. Paper T10 Managing Finances Wednesday 15 December 2010 The Association of Chartered Certified Accountants

Transcript of T10 Question

Page 1: T10 Question

Certified Accounting Technician ExaminationAdvanced Level

Time allowedReading and planning: 15 minutesWriting: 3 hours

This paper is divided into two sections:

Section A – ALL TEN questions are compulsory and MUST be attempted

Section B – ALL FOUR questions are compulsory and MUST be attempted

Do NOT open this paper until instructed by the supervisor.

During reading and planning time only the question paper may be annotated. You must NOT write in your answer booklet untilinstructed by the supervisor.

This question paper must not be removed from the examination hall.

Pape

r T1

0

Managing Finances

Wednesday 15 December 2010

The Association of Chartered Certified Accountants

Page 2: T10 Question

This is a blank page.The question paper begins on page 3.

2

Page 3: T10 Question

Section A – ALL TEN questions are compulsory and MUST be attempted

Please use the space provided on the inside cover of the Candidate Answer Booklet to indicate your chosen answer toeach multiple choice question.Each question is worth two marks.

1 A company has the following non-current assets:

20X5 20X6Non-current assets at closing net book value $200,000 $250,000

Depreciation for the 20X6 income statement is $30,000. No disposals were made in the period.

What is the correct figure for cash purchases of non-current assets during 20X6?

A $50,000B $80,000C $250,000D $20,000

2 An investment of $100,000 is made in a project. The scrap value is expected to be $15,000 at the end of the project.Four equal annual cash inflows of $35,000 will arise from the project, the first of which arises two years after theinitial investment.

What is the payback period and the accounting rate of return (based on initial investment) of the project?

Payback Accounting rate of returnA 3·9 years 11%B 3·9 years 28%C 2·9 years 11%D 2·9 years 28%

3 Which of the following statements is/are true with respect to investment appraisal methods?

(i) The accounting rate of return takes into account the timing of the cash inflows and outflows.(ii) Shareholders should benefit if a project is accepted which has a positive net present value.(iii) The internal rate of return calculation will always produce a unique answer.

A (i) and (ii)B (ii) onlyC (i) and (iii)D (ii) and (iii)

4 Which of the following would usually be considered to be the least liquid asset?

A Accounts receivableB Short-term investmentsC InventoryD Cash at Bank

3 [P.T.O.

Page 4: T10 Question

5 Company X has been offered a 2% discount if they pay their creditors within 10 days of the invoice. Payments areusually made after 30 days.

What is the compound annual cost of not taking the discount to the nearest percentage point?

A 24%B 28%C 45%D 27%

6 A company sells inventory for cash.

What will be the effect on the quick ratio (acid test) and the accounts receivable payment period?

Quick ratio Accounts receivable payment periodA Increase DecreaseB Decrease No changeC No change IncreaseD Increase No change

7 Which of the following is the first stage in a bankruptcy procedure?

A A trustee in bankruptcy is appointedB A bankruptcy order is grantedC A statutory demand for payment is issuedD A petition is made to the court

8 A company sells goods on credit and is expecting the following sales:

$March 20,000April 15,000May 25,000June 30,000

The following are the expected payments from accounts receivables:

50% in the month of sale30% one month after sale15% two months after sale5% are bad debts

What is the expected cash inflow in May?

A $18,250B $20,000C $19,375D $25,000

9 What is risk that can be diversified away known as?

A Systematic riskB Market riskC Unsystematic riskD Inherent risk

4

Page 5: T10 Question

10 A company is preparing a quotation for a project, based on relevant costing principles. The project will require 100kgof material X. The following information about material X is available:

Units already in Original cost price per kg Net realisable Current purchase price perinventory value per kg kg

50kg $6 $7 $8

The material is used frequently by the company.

What is the relevant cost of the material to be included in the quotation?

A $750B $700C $600D $800

(20 marks)

5 [P.T.O.

Page 6: T10 Question

Section B – ALL FOUR questions are compulsory and MUST be attempted

1 Mr Food owns a café and currently sells hot drinks and food such as sandwiches, crisps and cakes. Annual netincome is currently $200,000.

He wants to offer cooked meals and plans to extend his buildings and build a kitchen and a restaurant. In the newbuildings, Mr Food will be able to incorporate a kitchen which will meet the necessary hygiene standards and havea restaurant which will be able to seat up to 60 people. Architect’s fees of $8,000 have already been incurred indrawing up the plans and the building work is expected to take one year.

Building work

The total cost of building is estimated to be $200,000. This will be paid 25% at the beginning of the project and75% on completion of the building work, one year later. Depreciation will be charged over 25 years on a straight-linebasis.

The building work will cause disruption, which will cause some of the existing clients to leave. Mr Food estimates thatthe effect of this will be to reduce the current annual net income from the café by 10% for the duration of the buildingwork. Mr Food believes that the current annual net income from the café will return to 95% of its original level oncethe building work is completed, and will remain at this level.

Running costs (these will arise only when new operations commence in year two)

Cleaners will be employed costing $8,000 for each year the restaurant is open to diners.

Chefs will need to be employed, each earning $10,000 per year. The number of chefs employed will depend on theestimated number of weekly diners and will be calculated using the following table.

Number of weekly diners Number of chefs to be employed0–150 1151–250 2251–350 3351–450 4

The minimum number of chefs will be employed.

Waiting staff will be employed, costing $5,000 per year per member of waiting staff. The number of waiting staffrequired is estimated to be two in the first year the restaurant is open to diners, and then three in each subsequentyear.

Cash overheads are currently $30,000 per year. Mr Food estimates that the expansion will cause overheads toincrease by 8% in the first year the restaurant is open to diners and that they will then continue at this level.

Net income from the new operations (i.e. revenue less food costs)

People who frequent the restaurant Friday–Sunday are estimated to generate a net income of $10 per diner per day,whereas those frequenting the restaurant Monday–Thursday are estimated to generate a net income of $7 per dinerper day.

The estimated number of diners per week:

Year Friday–Sunday Monday–Thursdayper day per day

2 40 203 50 304 60 355 60 35

6

Page 7: T10 Question

Required:

(a) Using the discount tables provided, calculate the net present value of the restaurant project over a five-yearperiod. On the basis of your calculation conclude whether the expansion should take place (assume 52 weeksin a year). Ignore tax in your calculation. (12 marks)

(b) Briefly explain what a relevant cash flow is. Illustrate your points with examples from part (a). (5 marks)

(c) Explain how inflation affects the required rate of return of the investor, illustrating your answer with anumerical example. (3 marks)

Discount factor table extractsTime Factor 10%1 0·9092 0·8263 0·7514 0·6835 0·621

Annuity factor table extractsTime Factor 10%1 0·9092 1·7363 2·4874 3·1705 3·791

(20 marks)

7 [P.T.O.

Page 8: T10 Question

2 Expand Co’s production has suddenly increased from 10,000 units per annum to 50,000 units per annum. This risein production was not planned, but arose due to an increase in demand when a competitor went out of businessunexpectedly. Inventory of finished goods remain negligible. Due to the unplanned nature of the increase, inputs havehad to be sourced from many different suppliers at different prices and this has resulted in reduced productionefficiency. Production levels are expected to stay at 50,000 units per annum for the foreseeable future, and the ownersare concerned that they need to control working capital. They are specifically concerned about raw material inventory.

Required:

(a) The owners have heard about a just in time inventory management system (JIT).

(i) Explain the concept of JIT inventory management; (2 marks)

(ii) State three requirements for JIT inventory management to operate; (3 marks)

(iii) Advise, with reasons, whether or not JIT inventory management would be suitable for Expand Co in thecircumstances outlined above. (2 marks)

(b) Expand Co is keen to source the main raw material component from a single supplier (Wam Co). Two units ofthe raw material component are required for each unit of final output. Wam Co is willing to supply Expand Coand the following information is available:

The cost is $1·50 per unit, but a discount of 5% is offered on orders of 20,000 units or more. Expand Coestimates that the ordering costs are $300 per order and the holding cost of one item for one year will be 20%of the purchase price.

Calculate the order size to minimise total costs. Clearly show all workings.

Note: the economic order quantity is given by the formula

(10 marks)

(c) State three factors, other than price, that should be considered before selecting a new supplier. (3 marks)

(20 marks)

8

EOQC D

CO

H

=2

Page 9: T10 Question

3 Bake Co is a family owned company that makes and sells homemade cakes and confectionery under its own brandname. Its main customers are local supermarkets and shops. Bake Co has built up a reputation for quality, andrevenue has increased over recent years to $2 million per year. This increase in sales revenue is expected to continue,and the family are keen to expand and install new equipment in the factory. The cost of re-equipping the factory willbe high, estimated to be possibly as great as 25% of the present value of the company. The company has no debt.

Required:

Explain each of the following methods of raising finance, and discuss the usefulness of each for Bake Co’sexpansion plans.

(a) Overdraft (4 marks)

(b) Venture capital (4 marks)

(c) Term loan (4 marks)

(d) Equity (4 marks)

(e) Trade credit (4 marks)

(20 marks)

9 [P.T.O.

Page 10: T10 Question

4 Joe, a fisherman, lives in a coastal village which has recently become a favourite with tourists. He is considering givingup fishing and converting his boat to take tourists on coastal tours. Each tour will last approximately 45 minutes. Heis aware of the uncertain nature of the tourism business and is concerned that he may not have enough clients inyears with poor weather, to break even.

Based on information obtained from similar tour providers, Joe estimates that the following data will apply to thisbusiness:

1. The average number of tourists taken on trips in a year is 15,000.2. The average total costs incurred in a year when 10,000 tourists were taken on trips was $45,000.3. The average total costs incurred in a year when 25,000 tourists were taken on trips was $67,500.4. Fee per tourist is $4·00.

Required:

(a) (i) Calculate the breakeven point (number of tourists) and margin of safety (using the average number oftourists taken); (7 marks)

(ii) In an average year, Joe makes a profit of $6,000 from fishing. How many tourists would Joe need totake on trips in order to make the same profit? (2 marks)

(iii) Comment on your calculations in (i) and (ii) from the point of view of margin of safety. (2 marks)

(b) Using the graph paper provided illustrate your results from (a)(i) and (a)(ii) on a profit-volume chart. Clearlylabel the axes, show the fixed costs, breakeven point and profit of $6,000. (4 marks)

(c) Joe is concerned that due to the increased popularity of the village and the subsequent rise in private boatsowned by holiday makers using the harbour, the costs of storing his boat safely when not in use will increasenext year. This would increase fixed costs by $7,000. Sales and variable costs would not change.

(i) Briefly explain how this would affect your profit-volume line and show the effect of this on your graph(no further calculations are necessary); (2 marks)

(ii) From your graph estimate the new breakeven point and briefly interpret this for Joe (no furthercalculations are necessary). (3 marks)

(20 marks)

End of Question Paper

10

Page 11: T10 Question

Certifi ed Accounting Technician ExaminationAdvanced Level

The Association of Chartered Certifi ed Accountants

Managing Finances

Wednesday 16 June 2010

Time allowed

Reading and planning: 15 minutesWriting: 3 hours

This paper is divided into two sections:

Section A – ALL 10 questions are compulsory and MUST be attempted

Section B – ALL FOUR questions are compulsory and MUSTbe attempted

Do NOT open this paper until instructed by the supervisor.

During reading and planning time only the question paper may

be annotated. You must NOT write in your answer booklet until

instructed by the supervisor.

This question paper must not be removed from the examination hall.

Pape

r T1

0

Page 12: T10 Question

2

Section A – ALL TEN questions are compulsory and MUST be attempted

Please use the space provided on the inside cover of the Candidate Answer Booklet to indicate your chosen answer to each multiple choice question.Each question in this section is worth 2 marks.

1 If a company regularly fails to pay its suppliers by their normal due dates, it may lead to the following:

(i) Reduction in credit rating (ii) Trade receivables days may increase (iii) Diffi culty in obtaining credit from new suppliers (iv) Having insuffi cient cash to settle trade payables

Which TWO of the above could arise as a result of exceeding suppliers’ credit terms?

A (iii) & (iv) B (ii) & (iii) C (i) & (iv) D (i) & (iii)

2 Quilt Co grants credit terms of 30 days net to customers but offers an early settlement discount of 1% for payment within seven days.

What is the compound percentage cost of the discount to Quilt Co, to the nearest per cent?

A 17% B 18% C 19% D 20%

3 The following information relates to the inventory of Swiss Co:

Maximum Minimum Usage levels 1,000 kg per day 300 kg per day Lead times 10 days 5 days

Inventory can only be purchased in batches of 1,000 kg.

What is the maximum amount of inventory that should be held?

A 10,000 B 9,500 C 1,500 D 8,500

4 ‘An unconditional order in writing to pay the addressee a specifi ed sum of money either on demand or at a future date.’

What does the above defi nition describe?

A Certifi cate of deposit B Loan stock C Bill of exchange D Convertible bond

Page 13: T10 Question

3 [P.T.O.

5 Mint Co is considering using the services of a factor. Its annual sales are $5,000,000 with customers taking an average of 60 days to pay. The cost of fi nancing these is currently $65,753 per annum.

The factor will advance 70% of Mint Co’s sales invoices at a promotional rate of 8% of amounts advanced, the same rate that the company pays on its overdraft. In addition, the factor will reduce the average receivables period to 30 days. It will also take over the administration of the sales ledger, for which it will charge an annual fee of 1% of sales.

What is the cost or benefi t of employing the services of the factor compared to continuing without the factor?

A $17,124 cost B $17,124 benefi t C $15,753 benefi t D $15,753 cost

6 Which of the following statements are true?

Statement 1: A Central Bank deals in government debt. Statement 2: A Central Bank acts as lender of the last resort.

A Only statement 1 B Only statement 2 C Both statements 1 & 2 D Neither statement 1 nor statement 2

7 Which of the following relate to a fi nance lease?

A The lessor always supplies the equipment directly to the lessee B The lessor is responsible for servicing and maintaining the leased equipment C The period of the lease is usually shorter than the asset’s expected useful economic life D The lease has a primary period which is often followed by an indefi nite secondary period

8 Which of the following statements are true?

Statement 1: Being listed on a recognised stock exchange is a disadvantage for a company that wants to facilitate its growth by acquisition.

Statement 2: Being listed on a recognised stock exchange can enhance a company’s image.

A Only statement 1 B Only statement 2 C Both statements 1 & 2 D Neither statement 1 nor statement 2

9 Exactly one year ago, $100 of treasury stock was issued with a coupon rate of 4%, redeemable at par two years after issue. The stock currently has a market value of $102·25.

What is the stock’s interest yield, calculated to the nearest 2 decimal places?

A 3·91% B 4·00% C 1·71% D 2·20%

Page 14: T10 Question

4

10 A company has determined that the net present value of an investment project is positive $24,800 when using a 5% discount rate and negative $12,400 when using a 10% discount rate.

What is the internal rate of return of the project, to the nearest 1%?

A 6% B 7% C 8% D 9%

(20 marks)

Page 15: T10 Question

5 [P.T.O.

This is a blank page.

Section B begins on page 6.

Page 16: T10 Question

6

Section B – ALL FOUR questions are compulsory and MUST be attempted

1 Tastee Co is a small private company, which provides residential cookery courses to individuals. The company is currently collating information for its budget for the six months ending 31 December 2010. The following information is available:

(i) Provisional bookings for courses

Jul Aug Sep Oct Nov Dec Number of courses 0 2 4 3 4 4 Average number of attendees per course 0 16 8 16 8 12

No courses are run during the month of July. Tastee Co requires customers to confi rm their attendance in the week prior to the course, giving their credit card number as a guarantee. On average, 75% of provisional bookings are confi rmed and therefore result in fee payment.

(ii) Course prices

Each course lasts for three days, running from Friday to Sunday, and the fee for each course is $1,200 per attendee. All courses have to be paid for in full on the fi rst day of the course. Any returning customers are given a 25% discount on the fee. One in three of confi rmed bookings is from a returning customer.

(iii) Personnel costs

Tastee Co’s courses are delivered by experienced chef, Oliver James. He is not a member of staff at Tastee Co, but rather an outside contractor. Oliver James charges Tastee Co $3,000 for each course and is supported by his assistant, Mark White. Oliver James pays his assistant a wage of $500 per course.

Tastee Co has several part-time support staff. Wages costs for July are only $1,000 per month, but every month thereafter they amount to $4,000. However, the company is planning to increase pay by 5%, from September. All staff are paid at the end of each month for that month’s work.

(iv) Property costs

The business property comprises a small hotel, which includes an offi ce and large kitchen. Tastee Co rents the hotel at a rental cost of $24,000 per annum, with rents being paid quarterly in advance on 31 March, 30 June, 30 September and 31 December. However, the company has just received a notice from the landlord of the hotel, informing them that, with effect from 31 December 2010, annual rental payments will increase by 5%.

(v) Food costs

All food purchased by the hotel is organic and delivered weekly by Farm Fresh, a local fi rm. On the 5th day of each month, Tastee Co pays the bill for the previous month’s food deliveries. However, in December, Tastee Co is also required to settle its bill for December’s food requirements due to the two-week holiday taken by Farm Fresh in January. Over recent months, the cost for food on each residential course (including ingredients used for cooking) has been $50 per attendee. However, due to lower availability of organic produce, infl ation of 2% per month is expected to occur from September onwards. Food costs for June 2010 are expected to be $1,100 in total.

(vi) General Overheads

Tastee Co’s business rates are $8,350 per annum, paid in ten equal instalments from April to January each year. The company’s fuel cost of $4,200 per annum is paid for by equal monthly direct debits. However, it has just come to the fuel supplier’s attention that the direct debit failed to leave Tastee Co’s bank accounts in the months of January to April inclusive, due to a bank error. Therefore, the direct debit payment for July will include the amounts owed for these months too.

Page 17: T10 Question

7 [P.T.O.

(vii) Capital expenditure

Tastee Co needs to replace its three ovens in August. The list price of these is $1,800 each but Tastee Co’s Managing Director is confi dent that he can obtain a 10% discount on this price. The supplier is also allowing Tastee Co to pay for the ovens in two instalments – one in August and one in October.

(viii) Bank account

The balance on the company’s bank account is expected to be zero on 30 June 2010; it currently has a suffi cient overdraft facility to cover any cash defi cit arising.

Required:

(a) Prepare a cash budget for each of the six months in the period ending 31 December 2010.

Note: All workings should be rounded to the nearest $. (17 marks)

(b) Briefl y explain three motives for holding cash, as identifi ed by Keynes. (3 marks)

(20 marks)

Page 18: T10 Question

8

2 Bulb Co is a company specialising in the manufacture of a large range of light bulbs. Over recent years, the company has struggled to survive, having made a loss for the last two accounting periods ended 30 November 2008 and 30 November 2009. Despite this, the company currently has no debt fi nance at all, and it wholly owns all of its land and buildings, which are of substantial value. Over the last six months, business has begun to improve steadily, with an increase in sales of 20% compared to the same six month period last year. This improvement, however, has given rise to a need for increased working capital. The company has very little cash at present, having used most of its resources to keep afl oat during its loss-making period. The company therefore needs to accurately ascertain its working capital requirements for the next year. The following forecast fi gures are available.

Turnover for the year $65,000,000 Costs as a percentage of sales: Direct materials 20% Direct labour 25% Variable overheads 15% Fixed overheads 18% Selling and distribution costs 5%

The following average time periods are expected:

(i) Inventories

Raw materials are held in inventory for eight weeks and fi nished goods are held for six weeks. All fi nished goods inventories and work-in-progress inventories include direct materials, direct labour and variable overhead costs. Assume that goods remain in work-in-progress for two weeks and are 75% complete as regards direct materials and 50% complete as regards direct labour and variable overheads.

(ii) Accounts receivable

All customers are credit customers and take an average of nine weeks to pay.

(iii) Accounts payable

The lengths of credit taken vary depending on the category of expense. They are as follows:

Direct materials 6 weeks Direct labour 1 week Variable overheads 8 weeks Fixed overheads 5 weeks Selling and distribution costs 4 weeks

Required:

(a) Calculate the working capital requirement of Bulb Co for the next year. Assume that there are 52 working

weeks in the year. (12 marks)

(b) Bulb Co has decided that it should apply to the bank for an overdraft or a short-term loan to cover the increase in its working capital requirements over the next year. Having run its business with very little debt for many years now, it is unsure what factors the bank will take into account when deciding whether to lend to the company.

Describe FOUR factors that the bank will consider when deciding whether to grant this loan or overdraft to

Bulb Co. (8 marks)

(20 marks)

Page 19: T10 Question

9 [P.T.O.9

3 King Edward’s Hospital is a private hospital providing a range of care to patients. It is currently appraising a major capital investment project. The hospital directors are considering opening up a specialist fertility department in a wing of the hospital that is currently unused. If the decision is made to proceed with the investment, the equipment would be bought and the staff would be recruited immediately. Treatment and testing of patients could then start straight away.

In assessing the viability of capital projects, the hospital currently uses a target accounting rate of return of 20% (based on the average investment over the period) and a target payback period of four years. It will undertake a project only if BOTH the accounting rate of return and the payback period meet or exceed the targets. The following data are available for this proposed investment.

$’000 Cost of specialist equipment 8,000 Annual increased revenues 5,000 Annual increased staff costs (1,500) Annual increased other costs* (1,000)

* No depreciation is included in these fi gures.

After fi ve years, it is thought that most of the equipment would have become outdated and would have a residual value of only $0·5 million.

The company’s cost of capital is 10% per annum.

Required

(a) Briefl y defi ne both of the hospital’s current methods of project appraisal. (2 marks)

(b) Calculate the accounting rate of return and the payback period for the project and recommend on a purely

fi nancial basis whether the project should proceed. (8 marks)

(c) Briefl y outline TWO disadvantages of each of these two methods. (4 marks)

(d) You have mentioned to the directors that the net present value method is another valuable method of investment appraisal and that it may give a more accurate representation of whether the project should proceed. The directors have never heard of this method.

(i) Briefl y explain the net present value method; (2 marks)

(ii) Using the extracts from the discount and annuity tables provided, recommend whether the project should

proceed using this basis of investment appraisal. Show all workings clearly. (4 marks)

Extracts from discount factor tables Extracts from present value of annuity tables at a discount rate of 10% at a discount rate of 10%

Time Factor Time Factor 1 0·909 1 0·909 2 0·826 2 1·736 3 0·751 3 2·487 4 0·683 4 3·170 5 0·621 5 3·791 6 0·564 6 4·355 7 0·513 7 4·868 8 0·467 8 5·335 9 0·424 9 5·759 10 0·386 10 6·145

(20 marks)

Page 20: T10 Question

10

4 Click Co is a photographic printing company. It prints photographs for customers, mainly the general public, generating sales revenue of $1,800,000 per annum. All of the company’s printing machines are hired rather than owned outright. The current rate of rejects (i.e. prints of unacceptable quality) is 10%.

Click Co is now considering replacing the machines with more technically advanced ones, which would eliminate rejects altogether. The company’s standard costs for one photograph are as follows:

$ $ Direct materials – Paper: 0·04 – Ink: 0·03 ––––– 0·07 Direct labour 0·01 Variable overheads 0·02 ––––– Cost per unit produced 0·10 Cost of rejects 0·01 ––––– Variable cost per unit sold (good units) 0·11 –––––

Currently, the company sells 12,000,000 photos per annum. The company’s fi xed overheads are $35,000 per month. If the new machines were hired to replace the old machines, machine rental costs would increase by $5,000 per month. However, the new machines would reduce variable overheads by 30%. The ink used by the new machines is 10% cheaper than the ink being currently used.

Required:

(a) Calculate the current annual break-even point in good units. (3 marks)

(b) Calculate the annual break-even sales revenue if the new machines are hired. (5 marks)

(c) Calculate the annual level of good quality photos that would have to be demanded and sold for Click Co to

become indifferent about using the old or the new machines. Briefl y explain the basis of your calculation.

(6 marks)

(d) Briefl y outline THREE advantages and THREE disadvantages of using marginal costing, as compared to

absorption costing. (6 marks)

(20 marks)

End of Question Paper

Page 21: T10 Question

Certified Accounting Technician ExaminationAdvanced Level

Time allowedReading and planning: 15 minutesWriting: 3 hours

This paper is divided into two sections:

Section A – ALL 10 questions are compulsory and MUST be attempted

Section B – ALL FOUR questions are compulsory and MUST be attempted

Do NOT open this paper until instructed by the supervisor.

During reading and planning time only the question paper may be annotated. You must NOT write in your answer booklet untilinstructed by the supervisor.

This question paper must not be removed from the examination hall.

Pape

r T1

0

Managing Finances

Wednesday 16 December 2009

The Association of Chartered Certified Accountants

Page 22: T10 Question

Section A – ALL 10 questions are compulsory and MUST be attempted

Please use the space provided on the inside cover of the Candidate Answer Booklet to indicate your chosen answer toeach multiple choice question.Each question in this section is worth 2 marks.

The following information relates to questions 1 & 2

Annual sales $2,500,000

Costs as a percentage of sales:Raw materials 10%Direct labour 15%Production overheads 5%

Working capital statistics:Average raw materials holding period 4 weeksAverage finished goods holding period 2 weeksAverage receivables collection period 6 weeksAverage payables collection period 8 weeks

All finished goods values include raw materials, direct labour and production overheads. Assume there are 52 weeks inthe year.

1 How much working capital is required to finance finished goods inventory, to the nearest $?

A $28,846B $9,615C $57,962D $24,038

2 How much working capital is required to finance receivables?

A $384,615B $86,538C $115,385D $288,462

3 Which of the following are ALL money market instruments?

A cheques, certificates of deposit, depositsB bills, certificates of deposit and cashC bills, certificates of deposit and depositsD cash, cheques and bills

4 Assumption 1: Amounts of cash required in future periods can be predicted with certainty.

Assumption 2: The opportunity cost of holding cash is known and it does not change over a period of time.

Which of the above, if either, is an assumption on which Baumol’s model of cash management is based?

A BOTH Assumption 1 and Assumption 2B Assumption 1 ONLYC Assumption 2 ONLYD NEITHER Assumption 1 NOR Assumption 2

2

Page 23: T10 Question

5 Which of the following statements about systematic and unsystematic risk is correct?

A NEITHER systematic NOR unsystematic risk can be diversified awayB ONLY systematic risk can be diversified awayC ONLY unsystematic risk can be diversified awayD BOTH systematic AND unsystematic risk can be diversified away

6 A company purchases a non-current asset with a useful economic life of ten years for $1·25 million. It is expected togenerate cash flows over the ten year period of $250,000 per annum before depreciation. The company chargesdepreciation over the life of the asset on a straight-line basis. At the end of the period it will be sold for $250,000.

What is the accounting rate of return for the investment (based on average profits and average investment)?

A 20%B 15%C 33%D 25%

7 Bee Co is deciding whether to make components X and Y or buy them in from an outside supplier. The supplier wouldcharge $50 per unit of X and $55 per unit of Y.

Production cost ($ per unit) X YDirect material 12 13Direct labour 25 27Variable overhead 8 7Fixed overhead 7 6

Which of the components, if any, should be bought in from outside.

A X ONLYB Y ONLYC BOTH X and YD NEITHER X nor Y

8 A company has material B in its inventory, which it purchased in error at a cost of $800. The company is decidingwhether to: (i) use it as a substitute for material A which would cost $500 to buy in; or (ii) sell material B for $510cash LESS selling costs of $20; or (iii) use it in another contract.

If the company decides to use the material in another contract, what is its relevant cost?

A $800B $510C $500D $490

3 [P.T.O.

Page 24: T10 Question

9 Statement 1: Simple payback period takes into account the time value of money and uses cash flows rather thanprofits.

Statement 2: Internal rate of return takes into account the time value of money and uses cash flows rather thanprofits.

Which of the above statements is/are true?

A Statement 1 onlyB Statement 2 onlyC Both statement 1 and statement 2D Neither statement 1 nor statement 2

10 Statement 1: Positive covenants are promises by a borrower to do something.

Statement 2: Quantitative covenants are promises to keep within financial limits set by the lender.

Which of the above statements is true/false?

Statement 1 Statement 2A False TrueB False FalseC True FalseD True True

(20 marks)

4

Page 25: T10 Question

Section B – ALL FOUR questions are compulsory and MUST be attempted

1 Choc Co is a recently established company in the confectionery business. It has a keen and flexible workforce. Itmakes a range of organic chocolate bars. Its main customers are supermarkets, petrol stations and newsagents. Thepurchase patterns of Choc Co’s customers vary slightly. The supermarkets tend to order large amounts at varying timesthroughout the year. The petrol stations make regular weekly orders for smaller amounts and the newsagents have noregular pattern for their order size or frequency at all. The company’s chocolate bars are becoming increasinglypopular, with demand peaking at 5,000,000 bars over the last year.

The two key ingredients for Choc Co’s products are cocoa and sugar. Both of these ingredients are bought fromdifferent suppliers. Over the last year, the company has used 100,000 kg of sugar for its production. It currently orders5,000 kg of sugar at a time, at regular intervals throughout the year. The cost of placing an order is $35 and the costof storing 1 kg of sugar is $0·20 per annum. Choc Co does not hold any safety inventories of sugar since it has alwaysfound its supplier to be very quick and reliable when Choc Co places an order.

Cocoa is the more expensive of the two ingredients; therefore Choc Co’s purchasing policy in relation to this hasalready been established. It is bought from a very reliable supplier with whom it has good relationships. The currentordering policy for cocoa (i.e. total ordering and holding costs) costs the company in the region of $10,000 perannum.

Note: Economic order quantity is given by the formula:

Required:

(a) Calculate the annual cost of the current ordering policy for sugar. Ignore finance costs. (2 marks)

(b) Calculate the annual cost if the economic order quantity is used to determine the optimum order size forsugar. Ignore finance costs. (3 marks)

(c) Describe three costs associated with running out of sugar. (3 marks)

(d) Briefly explain how a just-in-time system for inventory procurement works and its effect on inventory levels.Briefly discuss whether or not it would be suitable for Choc Co’s business. (6 marks)

You are provided with the following further information, relating to part (e) only.

Choc Co pays $0·75 for each kilogram of sugar. Choc Co’s suppliers provide 60 days’ credit, which Choc Co alwaystakes but never exceeds. Its suppliers have now offered Choc Co a discount of 2% for payment within 14 days.

Choc Co has a short-term cost of finance of 10% per annum.

(e) Calculate the net gain or loss from accepting the discount. (6 marks)

(20 marks)

5 [P.T.O.

EOQC D

CO

H

=2

Page 26: T10 Question

2 Motor Co has recently developed a new environmentally friendly motor home. It has prepared two sets of figures forits forecast for the next year since it is currently undecided on its production level and selling price. The company istrying to decide whether it is more profitable to produce 400 or 500 motor homes, since a clear link between priceand demand has been identified. An extract from the coming year’s forecast is as follows:

Units produced and sold 400 500$’000 $’000

Revenue 34,000 37,500Materials 13,600 17,000Labour 8,400 10,500Overheads (1) 9,000 10,500

Note

(1) Overheads include both a fixed and a variable element which can be calculated from the above data.

Required:

(a) Calculate the contribution per unit at each sales level. (4 marks)

(b) Calculate the break-even point in units at each sales level. (3 marks)

(c) Calculate the margin of safety as a percentage at each sales level and explain what the margin of safety is.(3 marks)

(d) Calculate the budgeted profit at each sales level. (2 marks)

(e) Using the graph paper provided, draw and label a break-even chart for the motor homes at a sales level of400 units, showing activity levels between 0 and 400 units. (6 marks)

(f) State four assumptions on which break-even analysis is based. (2 marks)

(20 marks)

3 Educate Co is a fast-growing company specialising in the provision of adult education. It currently has ten languageschools and three information technology schools. It is estimated that the education sector will grow by 15% over thenext five years. The company is keen to take advantage of the opportunities which are available and is seeking toraise funds to finance its growth.

Since the company is ‘highly geared’ its advisors have suggested that it should seek to raise funds through some sortof share capital issue, rather than a loan. Educate Co wants to know what kind of groups invest in share capital, otherthan private individuals seeking profit. As part of this process, Educate Co is considering whether it would bebeneficial to float the company on the stock exchange.

Required:

(a) Describe FOUR groups of institutional investors that specialise in providing capital in order to make a return.(8 marks)

(b) Explain FOUR reasons why a company might seek a stock exchange listing. (8 marks)

(c) Explain the meaning of the term ‘highly geared’ and why it is important to a company trying to raise extrafinance. (4 marks)

(20 marks)

6

Page 27: T10 Question

4 Clear Co specialises in the production of UPVC windows and doors. It is considering whether to invest in a newmachine with a capital cost of $4 million. The machine would have an expected life of five years, at the end of whichit would be sold for $450,000.

If the new machine were to be purchased, the existing machine could be either sold immediately for $250,000, orhired out to another company at a rental amount of $100,000 per annum, payable in advance, for three years. If themachine is hired out rather than sold, it will have no residual value at the end of the three year period. The existingmachine generates annual revenues of $8 million and its running costs are $840,000 per annum.

If the new machine is purchased, revenues are expected to increase by 20%. In addition to this, however, machinerunning costs are also expected to increase. Estimates have shown that, in the first year with the new machine,running costs will increase by 18%. In every subsequent year thereafter, running costs will continue to be 18% higherthan each previous year’s costs.

The company’s cost of capital is 10%. All workings should be in $’000.

Required:

(a) Using the discount tables provided, calculate the net present value of the proposed investment, over fiveyears and state with reasons whether the machine should be purchased. (10 marks)

(b) Calculate the internal rate of return of the investment. (5 marks)

(c) Explain what capital expenditure is and how it should be treated in the accounts of a business. (3 marks)

(d) Explain why control of capital expenditure is so important within an organisation. (2 marks)

Discount factor table extractsTime Factor Factor

10% 20%1 0·909 0·8332 0·826 0·6943 0·751 0·5794 0·683 0·4825 0·621 0·402

Annuity factor table extractsTime Factor

10%1 0·9092 1·7363 2·4874 3·1705 3·791

(20 marks)

End of Question Paper

7

Page 28: T10 Question

Certified Accounting Technician ExaminationAdvanced Level

Time allowedReading and planning: 15 minutesWriting: 3 hours

This paper is divided into two sections:

Section A – ALL TEN questions are compulsory and MUST be attempted

Section B – ALL FOUR questions are compulsory and MUST be attempted

Do NOT open this paper until instructed by the supervisor.

During reading and planning time only the question paper may be annotated. You must NOT write in your answer booklet untilinstructed by the supervisor.

This question paper must not be removed from the examination hall.

Pape

r T1

0

Managing Finances

Wednesday 10 June 2009

The Association of Chartered Certified Accountants

Page 29: T10 Question

This is a blank page.The question paper begins on page 3.

2

Page 30: T10 Question

Section A – ALL TEN questions are compulsory and MUST be attempted

Please use the Candidate Registration Sheet provided to indicate your chosen answer to each multiple choice question.Each question in this section is worth 2 marks.

1 Chelm Co makes and sells one product, X. It uses marginal costing for all internal reporting. Details relating to theproduction of one unit of X are:

$Sales price 20·00Direct material cost 4·00Labour cost 2·00Variable overhead 1·50

Fixed overheads amount to $40,000 for the period under consideration. Budgeted sales for the period are 5,000units.

What is the contribution per unit of X?

A $16B $14C $4·5D $12·5

2 Boom Co sells one product, Y, for $12 per unit. Material cost is $2·20 per unit, labour costs are $1·40 per unit andvariable overheads are $0·80 per unit. The company incurs fixed costs of $540,000 per annum.

Calculate how many units of Y, to the nearest whole unit, must be sold in order to break-even.

A 71,053B 55,102C 45,000D 64,286

3 Lane Co has budgeted sales of 226,000 units per annum. Its fixed costs are $440,000 per annum. It sells oneproduct at a selling price of $8·80 per unit. This product has a contribution of $2·20 per unit.

Calculate the margin of safety for Lane Co, to the nearest whole unit.

A 66,667B 214,000C 176,000D 26,000

4 1 An increase in the period of credit given by suppliers2 A decrease in the period of credit given by suppliers3 An increase in the period of credit given to customers4 A decrease in the period of credit given to customers

Which of the above will increase the length of the cash operating cycle?

A 1 & 3 onlyB 2 & 4 onlyC 2 & 3 onlyD 1 & 4 only

3 [P.T.O.

Page 31: T10 Question

5 The following statements have been made about the probable long-term effects of introducing a just-in-time systemof inventory management:

(i) Inventory holding costs increase(ii) Labour productivity improves(iii) Manufacturing lead times decrease

Which of the above statements is true?

A (i), (ii) and (iii)B (i) and (ii) onlyC (i) and (iii) onlyD (ii) and (iii) only

6 The following statements have been made about a bank’s rights in relation to its customers:

(i) The bank has the right to be repaid overdrawn balances on demand, except where the overdraft terms require aperiod of notice.

(ii) The bank can use the customers’ money in any legally or morally acceptable way that it chooses.(iii) A customer’s money must always be available for immediate withdrawal, irrespective of the terms of the deposit.

Which of the above statements is true?

A (i) and (ii) onlyB (i), (ii) and (iii)C (i) and (iii) onlyD (ii) and (iii) only

7 The net present value of a proposed project is a positive $56,000 at a discount rate of 10% and a negative $28,000at 20%.

What is the internal rate of return of the project, to the nearest whole percentage?

A 17%B 13%C 30%D 8%

8 The following data relates to a manufacturing company:

Average finished goods inventory $25,000Average raw materials inventory $15,000Average WIP $30,000Sales for the period $400,000Gross profit margin 25%

WIP is 50% complete as regards materials and conversion costs.

What is the average production period (work-in-progress) to the nearest whole day?

A 37 daysB 73 daysC 30 daysD 18 days

4

Page 32: T10 Question

9 The following statements have been made about the essential elements of a contract:

(i) All contracts need to be in a strict legal form in order to be binding(ii) Both parties must have intended the contract to be legally binding(iii) Offer and acceptance must both have taken place

Which of the above statements are true?

A (i), (ii) and (iii)B (i) and (iii) onlyC (ii) and (iii) onlyD (i) and (ii) only

10 Which of the following is a potential remedy when one party has breached a contract?

(i) Specific performance(ii) Termination(iii) Quantum merit(iv) Damages

A All of the aboveB (iv) onlyC (i) and (iv) onlyD (i), (ii) and (iv) only

(20 marks)

5 [P.T.O.

Page 33: T10 Question

Section B – ALL FOUR questions are compulsory and MUST be attempted

1 Painless Co is a pharmaceutical company making both ibuprofen based and paracetamol based pain relief tablets. Itis considering outsourcing its whole packaging operation. The following information is available:

1 A total of 170 staff would be made redundant immediately. All staff are currently graded from A to D, with grade A staff being the lowest paid. Details of the staff to be made redundant are shown below:

Grade No. of redundancies Salary per annum per employee$’000

A 130 16B 20 19C 15 32D 5 64

All redundant Grade A and B employees would immediately receive 70% of their annual salary; Grade C and Demployees would immediately receive 85% of their annual salary.

2 The packaging warehouse, which will no longer be required if the outsourcing goes ahead, is rented from a localcompany under a lease that still has five years to run. There is no clause for the Painless Co to terminate thelease early. The company paid a $100,000 lump sum at the start of the lease five years ago. Painless Co spreadsthe cost of this lump sum over the life of the lease, charging $10,000 each year as an expense in its IncomeStatement. In addition to this, the annual rental costs are $25,000 per annum, also an expense in the IncomeStatement. The company pays the rent annually in advance, the last payment having been made yesterday.Painless Co hopes to sub-let this warehouse for $30,000 per annum, but expects it to take one year before asuitable tenant is found. Rent will then be charged annually in advance.

3 The company owns the packaging factory. If this proposal goes ahead, the factory will either be sold or leasedimmediately. The company has already met a potential buyer for the factory who would pay $300,000 for itimmediately. However, the interested party would alternatively be prepared to lease the factory for a five-yearperiod at a rental of $55,000 per annum, payable annually in advance. The value of the factory, in present valueterms, at the end of the rental period would be $65,000.

4 Annual sales of the company’s paracetamol and ibuprofen tablets are expected to be 64,000 and 67,200respectively. These are expected to remain the same for the next five years.

5 The total costs (excluding labour, which is dealt with in note 1) of making one thousand boxes of paracetamoltablets and one thousand boxes of ibuprofen tablets are $7·80 and $7·50 respectively. Alternatively, therespective costs, per thousand, of buying the boxes in are $13 for paracetamol and $14·20 for ibuprofen.

6 The company’s cost of capital is 10%.

7 Assume that all cash flows occur at the end of each year, unless told otherwise.

8 Answers should be given in $’000s.

Required:

Calculate the net present value of the proposal to outsource the manufacture of the packaging to Chenway, usingthe discount table extracts provided, and conclude whether the proposal should go ahead.

Discount factor table extracts Annuity factor table extracts

Time Factor Time Factor10% 10%

1 0·909 1 0·9092 0·826 2 1·7363 0·751 3 2·4874 0·683 4 3·1705 0·621 5 3·791

(20 marks)

6

Page 34: T10 Question

2 Rich Co is a cash-rich company wanting to maximise the return on its cash balances over the next six months. Itcurrently has $5,000,000 cash which it is ready to invest. Cash movements over the next six months are expectedto be as follows:

$Cash inflow in quarter 1 2,000,000Cash outflow in quarter 1 4,000,000Cash inflow in quarter 2 2,500,000Cash outflow in quarter 2 5,500,000

Based on market information available, interest rates for the year ahead have been estimated for deposits of cash withvarying maturity periods. These are as follows:

Quarter 1 Quarter 2Annual Annualrates % rates %

Deposit period 3 month 9·4 9·66 month 9·7 9·8

Notes1 A ‘quarter’ is a three-month period.2 All of Rich Co’s cash inflows and outflows take place on the last day of the quarter. This means that any surplus

cash is available for investment from the first day of the next quarter.3 When cash is placed on deposit for the set period of three months or six months, it is available for withdrawal

on the last day of that quarter. It can be used to pay cash outflows immediately.4 The Quarter 1 annual rates are the interest rates available as of today, for cash placed on deposit for periods of

three months and six months. 5 The Quarter 2 annual rates are estimates of the annual interest rates for cash placed on deposit for periods of

three months and six months.

You should assume that the annual rates given are simple rates of interest, rather than compound rates. In addition,you should assume that any interest earned during the first quarter is not available for re-investment in the secondquarter.

Required:

(a) Prepare a cash forecast for each of the two quarters showing clearly the opening and closing balances at theend of each quarter. (4 marks)

(b) Calculate which deposits should be made in order to maximise income, clearly showing the incomereceivable from the different options. (8 marks)

(c) Rich Co’s parent company is considering centralising the treasury function.

Required:

Describe four advantages of having a centralised treasury department. (8 marks)

(20 marks)

7 [P.T.O.

Page 35: T10 Question

3 Long-term finance.

Required:

(a) Explain what a stock market is and how it operates. (6 marks)

(b) Briefly describe any well-known stock market. (2 marks)

(c) Describe the features of the following different types of securities:

(i) Loan stock; (3 marks)

(ii) Convertible bonds; (3 marks)

(iii) Ordinary shares; and (3 marks)

(iv) Warrants. (3 marks)

(20 marks)

4 Curtain Co is a small, family-run business, which makes made-to-measure curtains for its customers. Curtain Co doesnot deal directly with the general public. Its customers consist mainly of local furnishing stores. Its turnover for thenext year is forecast to be $600,000. Curtain Co is always overdrawn and pays interest on its overdraft at 8% perannum.

Once a pair of curtains has been made, an invoice is raised and sent with the curtains to the customer. Invoices aresettled by cheque for each pair of curtains often resulting in several cheques from the same customer each month. Itis now considering introducing a system, whereby it invoices customers at the end of each month for their totaldespatches that month, requesting payment by standing order or direct debit within thirty days. It is expected that allCurtain Co’s customers would take advantage of the full credit period, but not exceed it.

Curtain Co’s owners have a few concerns about the new system. They are not sure whether standing order or directdebit payments would be more suitable for the business. Also, they have concerns about the cost of offering increasedcredit as, at present, customers pay their invoices 14 days, on average, after the month of sale. The business currentlypays $1·80 for each cheque banked; it would expect to bank 2,000 cheques over the coming year if it does not makechanges in the payment method. Its bank does not charge it for receipts into its account by standing order or directdebit.

Required:

(a) Briefly outline the main features of:

(i) standing order payments; and (ii) direct debit payments;

clearly distinguishing between the two. Recommend which method would be more suitable for Curtain Co’sreceipts. (6 marks)

(b) Calculate:

(i) current total finance costs (cheques and interest); and(ii) new total finance costs (cheques and interest) if the new system is introduced.

Conclude, from these calculations, whether the new system should be introduced. (7 marks)

(c) Curtain Co is considering obtaining trade references for its potential new customers.

List five pieces of information that you would expect to be present in a trade reference and state why suchreferences may not be very reliable without further evidence being obtained to support them. (7 marks)

(20 marks)

End of Question Paper

8

Page 36: T10 Question

Certified Accounting Technician ExaminationAdvanced Level

Time allowedReading and planning: 15 minutesWriting: 3 hours

ALL FOUR questions are compulsory and MUST be attempted.

Do NOT open this paper until instructed by the supervisor.

During reading and planning time only the question paper may be annotated. You must NOT write in your answer booklet untilinstructed by the supervisor.

This question paper must not be removed from the examination hall.

Pape

r T1

0

Managing Finances

Wednesday 10 December 2008

The Association of Chartered Certified Accountants

Page 37: T10 Question

ALL FOUR questions are compulsory and MUST be attempted

1 Wicker Co manufactures chairs. It has recently been approached by one of its customers, Chill Co, a garden furnitureretailer, who has asked it to enter into a five-year contract to supply a particular range of chairs. The chairs will formpart of Chill Co’s new luxury collection that they plan to start selling in three months’ time. Production would thereforehave to commence as soon as possible.

There are three types of chairs in the collection – The Recliner (R), The Soother (S) and The Handy (H). Wicker Cohas collated the following information:

1 Chill Co estimates that it would purchase the following quantities of R, S and H over the five year period:

Year 1 Year 2 Year 3 Year 4 Year 5R 20,000 22,000 24,200 26,620 29,282S 25,000 27,500 30,250 33,275 36,603H 30,000 33,000 36,300 39,930 43,923

Wicker Co’s intention is not to hold any finished goods inventory at the end of each year.

2 Chill Co will pay $200 for each R purchased, $100 for each S and $70 for each H.

3 Wicker Co will need to use a high-tech machine to make the chairs. This could be purchased immediately at acost of $200,000 and would have no scrap value at the end of the five years. Alternatively, Wicker Co hasanother machine on site, for which it no longer has any use, which could be modified at an immediate cost of$75,000. This machine would then be equivalent to the high-tech machine in terms of its capacity to makechairs. This machine was bought two years ago at a cost of $300,000. If it is not used for the Chill Co contract,it will be sold immediately for $100,000. If modified, it would have no scrap value at the end of the five years.

4 Wicker Co would also use some additional existing machinery to carry out the work on the chairs. Depreciationof $45,000 per annum would be allocated to this contract.

5 Materials usage for each of the three chairs are as follows:

R S HWood X (m2) 1 2 3Fabric (m) 3 2 1

6 Materials costs are as follows:Wood X $14 per m2

Fabric $22 per m

Wicker Co already has 160,000m2 of wood X in inventory, which it ordered in error last month. Wood X cost$20 per m2. If Wicker does not use wood X for this contract, it will either be used immediately as a substitutefor Wood Y, which currently costs $13 per m2, or sold for $11 per m2.

Also, Wicker Co already has 294,000m of fabric in inventory. The fabric cost the company $20 per metre. If itdoes not use this fabric for the Chill Co contract, the fabric would be sold immediately for $10 per metre sinceit has no other use for it.

7 Each chair requires the following amount of time from Wicker Co’s skilled and unskilled labour forces:

R S HSkilled (hours) 2 2 2Unskilled (hours) 1 1 1

At present, the company’s skilled labour force costs $8 per hour and its unskilled labour force costs $6 per hour.The company’s skilled labour force is working to full capacity, so new staff will need to be recruited if the ChillCo contract is entered into. These will cost $9 per hour and will be needed for the whole period of the contract.However, there is an unexpected surplus capacity of unskilled labour for the first year. It is thought that existingemployees can provide 35,000 of the unskilled labour hours required in the first year, at no extra cost to thecompany. The remainder of the unskilled hours required will be provided by the employment of additional staffat $6 per hour.

8 Variable overheads will be incurred at the rate of $4 per skilled and unskilled labour hour used.

2

Page 38: T10 Question

9 Fixed overheads are expected to be $400,000 per annum. Of these, $220,000 relates to apportionment offactory rent. The factory space that would be used for the contract is currently unused and Wicker Co does notforesee it being used at all in the future unless this contract is entered into. The remainder of the fixed overheadcost relates to a new warehouse that would be rented to cope with the increased storage space required for theChill Co contract.

10 The company’s cost of capital is 11·5%.

Required:

(a) Within the context of relevant costing, explain how to determine the relevant cost of the following items forthe Chill Co contract:

(i) machine costs (note 3)(ii) wood costs (note 6)(iii) fabric costs (note 6)

Note: whilst you should explain the correct costs to be used, detailed calculations should be performed in (b)below. (8 marks)

(b) Using the discount factors provided below, calculate the net present value of the contract and recommendwhether Wicker Co should enter into the contract. Workings should be in $’000, to the nearest $’000.

Discount factors

Time Factor11·5%

1 0·8972 0·8043 0·7214 0·6475 0·580

(32 marks)

(40 marks)

3 [P.T.O.

Page 39: T10 Question

2 Brush Co is a recently established company specialising in the manufacture of a range of drugs for the pharmaceuticalindustry. Two brothers, Thomas and Gerald Broom, formed the company and have just finished the first year ofbusiness. Sales are made to customers on 60-day payment terms and all suppliers offer 30 days’ credit. All of theraw materials purchased by Brush Co only last for a limited time. Therefore, it is the company’s policy that suchchemicals are used within 75 days of purchase.

Whilst the brothers are experienced in the field of pharmaceuticals, they are finding it difficult to understand some ofthe financial matters associated with running a company.

You are employed in the company as an accounting technician and have collated the following information for the lastyear.

$Sales 1,500,000

Raw material purchases 378,000Direct labour costs 240,000Variable production overheads 215,000Apportioned fixed production overheads 185,000

Average receivables 356,000

Average inventories:Finished goods 210,000Work-in-progress (WIP) 58,000Raw materials 82,000

Average payables:Materials 45,000Variable and fixed overheads 75,000Direct labour 9,000

Other relevant information1 All finished goods inventory and WIP values include raw materials, direct labour, variable production overheads

and apportioned fixed production overhead costs. 2 Assume WIP is 70% complete.3 Assume there are 365 days in one year.4 Assume that production and sales volumes are the same.5 The length of the average working capital cycle in this type of business is 90 days.

Required:

(a) Calculate Brush Co’s working capital cycle (cash operating cycle) in days. All workings should be rounded tothe nearest complete day. (10 marks)

(b) Explain what working capital management is and why it is important. (4 marks)

(c) From your calculations in part (a), identify possible concerns that Brush Co’s chief accountant may haveabout the company’s working capital cycle. (6 marks)

(20 marks)

4

Page 40: T10 Question

This is a blank page.Question 3 begins on page 6.

5 [P.T.O.

Page 41: T10 Question

3 Print Co is a printing company providing a range of printing services to commercial businesses and the public (non-commercial customers.) It has three main suppliers, which it uses for the majority of its supplies, but also uses anumber of smaller suppliers for its sundry purchases. Whilst commercial customers are sometimes provided with acredit facility, non-commercial customers must pay on the day with cash, cheque, debit or credit card. Print Coprepares cleared funds forecasts on a weekly basis.

You are an accounting technician for the company and have been asked to prepare a cleared funds forecast for theperiod Monday 19 January to Friday 23 January 2009 inclusive. You have been provided with the followinginformation:

1 Receipts Commercial customers Month to:Customer Credit terms Payment 19 Jan. 19 Dec. 19 Nov.name method 2009 sales 2008 sales 2008 salesAnchor Co 2 calendar months Cheque $18,000 $24,000 $16,000Beauty Co 1 calendar month Cheque $13,000 $18,000 $17,000Kent Co 1 calendar month BACS $20,000 $18,000 $15,000Hut Co None Cash $2,200 $5,400 –Light Co None Cheque $4,500 – $2,200

Non-commercial customersPayment Mon Tue Wed Thu Fri Mon Tue Wed Thu Frimethod 12 13 14 15 16 19 20 21 22 23

Jan. Jan. Jan. Jan. Jan. Jan. Jan. Jan. Jan. Jan.Cash $300 $230 – $120 – – $220 $350 – $430Cheque $220 – – – – – – – – –Debit card $255 $426 – – – $170 $210 – – –Credit card – – – – – – – – – $3,500

Notes(i) Receipt of money by BACS is instantaneous.(ii) All commercial customers are reliable, as cheques are always received on 19th day of each month. They

are banked on the same day and clear on the fourth working day following the date of receipt.(iii) Debit card payments are credited to Print Co’s account on the next working day following the date of sale.

They represent cleared funds as soon as they are credited.(iv) Credit card receipts clear in Print Co’s bank account on the fifth working day following the day of sale.(v) A ‘working day’ is a Monday, Tuesday, Wednesday, Thursday or Friday.

2 Payments to suppliersMain suppliersSupplier name Credit terms Payment Jan. 2009 Dec. 2008 Nov. 2008

method purchases purchases purchasesInk Co 1 month Direct debit $6,500 $5,500 $4,200Toner Co 2 months Direct debit $8,500 $8,000 $10,600Paper Co 1 month Direct debit $7,200 $10,500 $5,400

Notes(i) Print Co’s monthly direct debit payments will leave its account on 23 January. (ii) Print Co pays its sundry suppliers by cheque each month. These total $8,200 for January and will also be

sent out on 23 January.

3 Wages and salariesAll staff are paid a monthly salary by BACS on the 22nd day of each month. The total salaries cost for Januaryis $9,600. BACS transactions are initiated and paid on the same day.

4 Other paymentsEvery Monday morning, the cashier withdraws $150 from the company bank account to use as petty cash. Themoney leaves Print Co’s bank account straight away.

6

Page 42: T10 Question

5 Other informationThe balance on Print Co’s bank account will be $35,000 on 19 January 2009. This represents both the bookbalance and cleared funds.

Required:

Prepare a cleared funds forecast for the period Monday 19 January to Friday 23 January 2009 inclusive usingthe information provided. Show clearly the uncleared funds float and the total book balance carried forward eachday.

(20 marks)

4 Slim Jim Co is a five-year old private company specialising in the manufacture of a range of health drinks, foods andsupplements aimed at the fitness market. At present, their biggest customers are health food shops and fitnesscentres. However, now that their brand has become established, the wealthy owners, who also manage the business,are convinced that sales could be increased dramatically through the opening of an internet shop. They are currentlyconsidering how best to fund the expansion of the business.

Funds would be needed to set up the website, expand manufacturing at the factory, and employ more staff to dealwith administration, despatch and delivery of the web orders.

It is estimated that $2 million would be needed for the expansion. At present, the market value of the company’sequity is $4 million and the company has loans of $0·5 million, repayable in six months time. The company also hascash built up from retained earnings of $1·3 million.

Required:

(a) Outline THREE appropriate sources of medium/long-term finance that may be available to Slim Jim Co tofinance its expansion. (Presume that government grants and leasing are NOT appropriate.) (5 marks)

(b) Discuss FIVE factors that Slim Jim Co should take into account when deciding on the mix of debt and equityfinance. (15 marks)

(20 marks)

End of Question Paper

7

Page 43: T10 Question

Certified Accounting Technician ExaminationAdvanced Level

Time allowedReading and planning: 15 minutesWriting: 3 hours

ALL FOUR questions are compulsory and MUST be attempted.

Extract from discount factor tables and annuity factor tables are onpage 2.

Do NOT open this paper until instructed by the supervisor.

During reading and planning time only the question paper may be annotated. You must NOT write in your answer booklet untilinstructed by the supervisor.

This question paper must not be removed from the examination hall.

Pape

r T1

0

Managing Finances

Wednesday 11 June 2008

The Association of Chartered Certified Accountants

Page 44: T10 Question

Extract from discount factor tables at a discount rate of 10%

Time Factor1 0·9092 0·8263 0·7514 0·6835 0·6216 0·5647 0·5138 0·4679 0·42410 0·386

Extracts from annuity factor tables at a discount rate of 10%

Time Factor1 0·9092 1·7363 2·4874 3·1705 3·7916 4·3557 4·8688 5·3359 5·75910 6·145

2

Page 45: T10 Question

ALL FOUR questions are compulsory and MUST be attempted

1 Alan Web is a famous musician and composer. He has recently teamed up with a successful producer of musicals,David Daniels. Together, they are going to fund a new musical ‘The Fiesta’. The musical will run in Jamin (the capitalof Bundai) for five months. If it is successful, it will then tour the rest of Bundai for a further period.

Alan and David are each investing $100,000 in the production. They have opened a business bank account to beused solely for the musical’s income and expenses. The investment cash will be paid into this account on 30 June2008. Rehearsals will take place throughout the month of July, with the musical’s official opening night taking placeon 1 August.

Alan and David appreciate the need for cash forecasting and have made the following estimates for income andexpenses for the six months from 1 July to 31 December 2008:

1 Tickets for the musical will go on sale on 1 July. Tickets will be sold through an agent who is to be paidcommission by Alan and David. The price of tickets will vary according to the seat location. Prices are as follows:

Location of seat PriceStalls $30Front $20Rear $15

Ticket sales each month will include not only tickets for that month’s performances but also advance bookingsfor later months. Sales per month are expected to be as follows:

Jul Aug Sep Oct Nov DecStalls 10,000 20,000 No further stalls seats availableFront 30,000 50,000 20,000 No further front seats availableRear 10,000 30,000 40,000 50,000 45,000 35,000

The agent will pay the proceeds of ticket sales into Alan and David’s business account in the same month as thetickets are sold to the public. The agent will then invoice Alan and David for the commission it charges of 6%.This will be payable in the month following the ticket sales.

2 Sales from programmes, CDs and beverages are expected to be $150,000 in August and $175,000 for eachmonth thereafter. All costs relating to the production of these goods will be incurred in July. Costs represent 10%of sales value.

3 All of the staff in notes 4 and 5 below will only receive 50% of their usual monthly salary for the month ofrehearsals.

4 The leading lady, who plays the key role in the musical, will be paid a salary of $16,000 per month. Theremaining 20 cast members (actors/singers) will be paid a monthly salary of $8,000 each. The leading lady andthe cast members will be paid at the end of each month for that month’s work.

5 A live orchestra will also be playing every night. The orchestra also starts rehearsing at the beginning of July.There are 25 musicians in the orchestra, each of whom will be paid a salary of $4,000 per month. Musicianswill be paid at the beginning of each month for the previous month’s work.

6 Other production staff members are provided by an independent production company and are expected to cost$80,000 per month in total. They will not be required until the musical opens in August. They will be paid atthe beginning of each month for that month’s work. However, in addition to this monthly cost, there will also bea production company administration fee of 5%, payable monthly in arrears.

7 The production company charges a separate amount for the production equipment, also required at the beginningof August. This is expected to be $33,000 per month, payable on the first day of each month.

8 The costumes for the cast will be made in July. A deposit of $50,000 needs to be paid at the beginning of July,with the remaining balance of $25,000 to be paid at the end of that month.

9 The background for the stage is currently being painted by local artists. Its cost of $180,000 will be payableupon its completion at the beginning of July.

3 [P.T.O.

Page 46: T10 Question

10 The musical will be held at the National theatre, which must be hired for $600,000 per month, payable at thebeginning of each month. The theatre will be used for rehearsals in July as well.

11 Certain equipment, not supplied by the production company, costing $155,000 will be bought in July for theproduction. Depreciation for the six-month period will total $31,000.

12 All workings should be in $’000 to the nearest $’000.

Required:

(a) Prepare a monthly cash budget for the musical for each of the six months to 31 December 2008, showingthe cash balance at the end of each month. (18 marks)

(b) Alan and David are very aware that their actual cash flow in the six months ending 31 December 2008 coulddiffer greatly if the above estimates prove to be inaccurate. They have heard of ‘sensitivity analysis’ and want toknow more about it.

Required:

Briefly explain sensitivity analysis and discuss how it might best be used to make the cash budget for themusical more useful.

Note: no further calculations are required. (6 marks)

(c) Alan and David are considering whether they should set up their own company for future theatrical productions.They want to know whether, over a ten-year period, it would save money.

If they set up the company, they would avoid the following annual costs:

(i) Estimated external production staff costs of $900,000.(ii) Estimated administration fees of $45,000.(iii) Estimated production equipment costs of $400,000.

However, they would incur the following set-up costs immediately:

(i) Legal costs of $4,000.(ii) Company registration costs of $1,000.(iii) Equipment costs of $1,800,000.

A further amount of $1·5 million would be payable for the equipment in one year’s time.

The following annual costs would arise:

(i) Staff salaries of $500,000.(ii) Professional fees of $10,000.(iii) Equipment maintenance costs of $75,000.(iv) Depreciation costs of $200,000.

Alan and David will need to spend a considerable amount of time setting up the company. They estimate that,as a result of this, their revenue may be $75,000 lower in the first year than it would otherwise have been (ignoreagent’s commission). Their cost of capital is 10% per annum.

Required:

Using the discount tables provided, calculate the net present value (NPV) of the proposal to set up theproduction company, at the company’s cost of capital. Advise Alan and David whether they should set upthe company.

Note: show all workings in $’000. (10 marks)

4

Page 47: T10 Question

(d) Alan and David have been advised that the ‘pay-back period method’ may also be a useful way of assessingwhether to set up the production company. They know nothing about this method of project appraisal.

Required:

(i) Explain the simple payback period method;

Note: calculations are not required. (2 marks)

(ii) State two differences between the calculation of the simple payback period of a project as compared tothe calculation of the net present value of a project; (2 marks)

(iii) State two advantages of using the simple payback period method as compared to other methods ofproject appraisal. (2 marks)

(40 marks)

2 Light Co is a privately owned company specialising in the manufacture of lighting equipment. It supplies lighting tocustomers, who take an average of 30 days to pay. It has an overdraft on its current account of $2m. The compoundannual interest rate charged on this account is 12%, with interest being charged to the account daily.

In order to reduce its overdraft, Light Co is now considering introducing discounts to customers who pay within seven days.

Required:

(a) Calculate the maximum discount that Light Co should offer for payment within seven days if it wants to avoidany increase in its overall finance costs and explain the basis of your calculation. (4 marks)

(b) Briefly explain the difference between simple and compound interest rates, using Light Co’s overdraft interestas an example. (4 marks)

(c) One year later, despite introducing a tempting discount to customers, Light Co has found that very few customershave paid early and taken the discount. In fact, receivables days have increased significantly, as has thecompany’s overdraft. Light Co is therefore considering factoring its debts in the coming year.

Credit sales for the last year totalled $12 million, with average receivables of $2 million. Next year, sales areexpected to increase by 10%. Receivables days are expected to increase to 70 days if the factoring arrangementis NOT entered into. A factoring company has put forward the following proposal to Light Co:

(i) Receivables days will be reduced to 28 days as a result of stricter credit control procedures.(ii) The factor will charge interest of 13% per annum on the advances.(iii) The factor will charge an administration fee of 1·5% of turnover for the service.(iv) The factor will advance 80% of the value of sales invoices.

Should Light Co enter into the agreement, it will make its credit controller redundant. She earns a salary of$18,000 per annum. Current bank overdraft rates have remained the same at 12% per annum.

Required:

Evaluate whether it is financially viable for Light Co to factor its debts in the coming year. (12 marks)

(20 marks)

5 [P.T.O.

Page 48: T10 Question

3 The Kitchen Co is an innovation company set up two years ago by its key shareholder and director, Brian Geek. Itcurrently has a range of about two thousand kitchen products on the market, the most successful of which is a gadgetcalled the ‘Fish Eye’. This is a revolutionary utensil, the size of a kitchen knife, which plugs into the power supplyand is used on cooked fish to identify any fish bones that need to be removed prior to serving. The ‘Fish Eye’ explodedonto the market two years ago, as the company’s introductory product, and sales have continued to grow rapidly eversince.

One of Brian Geek’s friends has warned him about the high incidence of overtrading for new businesses selling highdemand, innovative products. Brian Geek is therefore concerned that The Kitchen Co’s financial position be carefullymonitored. Its turnover has increased by 100% over the last year, and its trade receivables and inventories havedoubled. The company’s current ratio has fallen over the last year from a ratio of 3:1 to a ratio of 2·5:1. The industrynorm is 2:1.

The company has a $1 million overdraft facility on its current account from its bank. Whilst the company has neverused even half of its limit, it often relies on the overdraft facility to finance its working capital.

Required:

(a) Explain the term ‘overtrading’. (2 marks)

(b) Describe the symptoms that MAY be present in a company that is overtrading. (5 marks)

(c) Briefly discuss whether The Kitchen Co is overtrading. (5 marks)

(d) One of the key components used to make the ‘Fish Eye’ is component X, which is imported from overseas. BrianGeek wants to manage his inventory levels of component X more efficiently. He wants to make sure that he canmeet demand for production and sales whilst at the same time avoiding excessive inventory levels. The followinginformation relates to component X:

Cost of component X $24 per unitUsage per day 1,000 unitsMaximum lead time 20 daysMinimum lead time 10 daysAverage lead time 15 daysCost of ordering $650 per orderHolding costs $2 per unit per annum

Usage per day is always constant. The re-order level is set at the maximum expected requirement in lead timeplus 25%.

Required:

(i) Calculate the re-order level; (3 marks)

(ii) Calculate the Economic Order Quantity (EOQ) using the following formula:

Where Co = the cost of placing one orderD = the annual demand in unitsCH = the cost of holding one unit per annum

Note: you should assume that there are 48 working weeks in the year and five working days in each ofthe weeks. (2 marks)

(iii) Calculate the maximum inventory level for component X using the information provided. (3 marks)

(20 marks)

6

EOQC D

Co

H

=2

Page 49: T10 Question

4 Banks and the money markets play an important role in a country’s economy.

Required:

(a) Explain the difference between retail banks and wholesale banks. (4 marks)

(b) Describe four functions of a central bank. (8 marks)

(c) Name the different money markets and briefly explain their role within the economy. (4 marks)

(d) Briefly describe four money market instruments. (4 marks)

(20 marks)

End of Question Paper

7

Page 50: T10 Question

Certified Accounting Technician ExaminationAdvanced Level

Time allowedReading and planning: 15 minutesWriting: 3 hours

ALL FOUR questions are compulsory and MUST be attempted.

Do NOT open this paper until instructed by the supervisor.

During reading and planning time only the question paper may be annotated. You must NOT write in your answer booklet untilinstructed by the supervisor.

This question paper must not be removed from the examination hall.

Pape

r T1

0

Managing Finances

Wednesday 12 December 2007

The Association of Chartered Certified Accountants

Page 51: T10 Question

ALL FOUR questions are compulsory and MUST be attempted

1 Robo Clean Co is a recently established innovation company. It currently has one product on the market, the‘Robovac’, a robotic floor cleaner. This has been extremely successful. The company is currently developing a newrobotic cleaner called ‘Robomum’ that vacuums, dusts and presses.

To date $120,000 has been spent on developing the product. The company has also incurred $250,000 of marketresearch costs, although the invoice for these costs has only just been received and will be paid in January.

Since the set-up costs are substantial, a final decision now needs to be made as to whether it is viable to manufactureand sell ‘Robomum’. The following revenues and costs have been estimated:

1. A new factory, to be used solely for the production of ‘Robomum’, will need to be built. This will take nearly ayear to build and is expected to cost $11·75 million in total, payable in two instalments. The first instalment of$6m will be paid at the start of the building work and the second instalment for the remaining balance will bepaid when the building work has been completed at the end of the year.

2. Robo Clean Co will immediately enter into a one-year contract with a project management company, who willoversee the building of the factory. The total cost of this during the year will be $250,000.

Two production lines will need to be installed in the factory at a further cost of $1,500,000 payable at the endof the build in one year’s time.

3. The machinery for the production of ‘Robomum’ also needs to be built-to-order and is expected to cost $2·5m,payable in one year’s time. Its terminal value is nil. Depreciation will be charged as soon as productioncommences (as soon as the build finishes in one year’s time) at 10% per annum on a straight-line basis.Maintenance costs for the machinery are estimated at $250,000 per annum.

4. Production and Sales will commence in the year following the build. Sales quantities and prices for ‘Robomum’are expected to be as follows:

Years 1 2 3 & 4 5 to 9(each year) (each year)

Sales volume (’000 units) 5 10 30 50Sales price ($) 1,000 800 700 500

It is anticipated that by the beginning of year 10, a new robotic helper will have replaced ‘Robomum’, hencethere will be no further sales.

5. Material costs for ‘Robomum’ are estimated at $125 per unit.

6. Labour costs are estimated at $100 per unit.

7. Fixed production overheads on the new factory are estimated at $240,000 per annum. Variable productionoverheads are expected to be $50 per unit.

8. Head office costs of $4·5m per annum will be allocated to ‘Robomum’ when production commences. Of thesecosts, only $3·7m is incremental.

9. The introduction of ‘Robomum’ is expected to adversely affect sales of ‘Robovac’. It is thought that, for every twounits of ‘Robomum’ sold, one unit of ‘Robovac’ will be lost. ‘Robovac’ is currently sold for $150 per unit andgenerates a net cash flow of $50 per unit.

10. The company’s cost of capital is 5%.

11. Assume that all cash flows occur at the end of the year, unless stated otherwise.

12. All workings should be in $’000, to the nearest $’000.

2

Page 52: T10 Question

Required:

(a) Using the discount tables provided, calculate the net present value (NPV) of the project at the company’scost of capital. Conclude as to whether Robo Clean Co should proceed with the project. (22 marks)

(b) Explain the main principles to differentiate between relevant and irrelevant costs for investment appraisal.Wherever possible, use the costs of ‘Robomum’ to illustrate your answer. (6 marks)

(c) Define and distinguish between capital and revenue expenditure. Explain whether the $250,000 per annumcosts of maintaining the machinery for ‘Robomum’ are capital or revenue in nature. (6 marks)

(d) Robo Clean Co is considering alternative ways of funding the project. The machinery is one of the largest costs.The company has heard of hire purchase agreements, finance leases and operating leases, but is not surewhether any of these will be suitable, given that the machinery has to be built-to-order.

Briefly describe each of the following

(i) Hire purchase agreements; (2 marks)

(ii) Finance leases; (2 marks)

(iii) Operating leases. (2 marks)

Present value table (extract)

Periods (n) Discount rate (r)5%

1 0·9522 0·9073 0·8644 0·8235 0·7846 0·7467 0·7118 0·6779 0·64510 0·614

Annuity factor table (extract)

Periods (n) Discount rate (r)5%

1 0·9522 1·8593 2·7234 3·5465 4·3296 5·0767 5·7868 6·4639 7·10810 7·722

(40 marks)

3 [P.T.O.

Page 53: T10 Question

2 All Weather Windows Co manufactures and fits windows for domestic customers. The company needs to forecast itsworking capital requirements for the year ahead. The following figures are available:

Sales revenue $7,600,000

Costs as percentage of sales revenueRaw materials 22%Direct labour 18%Variable production overheads 7%Apportioned fixed production overheads 12%Other costs 5%

Working capital statisticsAverage raw material holding period 6 weeksAverage work-in-progress (WIP) holding period 3 weeksAverage finished goods holding period 5 weeksAverage trade receivables’ collection period 2.5 weeksAverage trade payables’ payment period on:Raw materials 8 weeksDirect labour 2 weeksVariable production overheads 4 weeksFixed production overheads 6 weeksOther costs 3 weeks

Other relevant information1. All finished goods inventory and WIP values include raw materials, direct labour, variable production overheads

and apportioned fixed production overhead costs.

2. Assume WIP is 80% complete as to materials; 75% complete as to direct labour; 50% complete as to variableproduction overheads and fixed production overheads.

3. Assume there are 52 weeks in one year.

4. Assume that production and sales volumes are the same.

5. All workings should be in $’000, to the nearest $’000.

Required:

(a) Calculate the estimated average working capital required by All Weather Windows Co for the year, showingall necessary workings. (14 marks)

4

Page 54: T10 Question

(b) All Weather Windows Co approaches its bank to ask for a loan to assist with a new machinery purchase. Thebank refuses the loan on the basis that the company is too highly geared and its interest cover is too low. Theindustry averages for gearing and interest cover are as follows:

Industry average

GearingPrior-charge capital

x 100% 100%–––––––––––––––––Shareholders’ funds

Interest cover 3 times

The following figures are extracts from All Weather Windows Co’s accounts:

Long-term loans $6millionOrdinary share capital $3 millionReserves $1 millionProfit before interest $1.2millionInterest $500,000

(i) Calculate the company’s level of gearing (also known as leverage) using the same ratio as above, andexplain what it means when All Weather Windows Co is said to be ‘highly geared’. (3 marks)

(ii) Calculate the company’s interest cover and explain what it means when All Weather Windows Co is saidto have low interest cover. (3 marks)

(20 marks)

5 [P.T.O.

( )

Page 55: T10 Question

3 Cool Ski Co is a skiwear retailer operating through its website shop. It is run by its three directors/shareholders whostarted the business three years ago. Its busiest months of the year are December, January, February and March, withsales for the rest of the year being relatively insignificant.

In December the company prepares a cash budget for January, February and March. The following figures from itsprofit forecast for December 2007 through to March 2008 are currently available. However, they may need to berevised and should be read together with the notes below.

Dec Jan Feb Mar$’000 $’000 $’000 $000

Sales revenue (1) 450 650 750 350Purchases See notes 2 and 3Staff costs (4) 45 60 70 30Packaging costs (5) 7 10 12 6Distribution costs (6) 35 50 58 28Other costs (7) 50 75 85 55

Notes:1. The company does not provide any credit to customers. However, customers who join the company’s members’

club are given a 5% discount on all of their purchases. Half of customers are club members. The sales revenueforecasts above have been calculated before any discounts have been taken into account.

2. Purchases represent 40% of gross sales revenue. Sales revenue in November was $95,000. 3. Suppliers allow two months’ credit.4 All staff are paid at the beginning of the month for the previous month’s work.5. Packaging costs are paid one month after they are incurred.6. Distribution costs are paid in the month in which they are incurred.7. Other costs include depreciation of $12,000 per month. They also include rental costs of $30,000 per month,

which are paid quarterly in December, March, June and September. The remainder of ‘other costs’ are paid inthe month in which they are incurred.

8. The bank charges interest of 0·5% per month for the overdraft, calculated on the closing bank balance eachmonth, and payable in the following month.

9. The overdraft on Cool Ski Co’s bank account at 31 December 2007 is expected to be $500,000.10. All workings should be in $’000, to the nearest $’000.

Required:

(a) Prepare a monthly cash budget for each of the three months to 31 March 2008, showing the cash balanceat the end of each month. (10 marks)

(b) Cool Ski Co is considering expanding its business. It wants to branch out into the manufacture of its own brandof skiwear, and then expand its customer base to include wholesale customers, such as high street retailers. Thethree directors/shareholders have produced a business plan and are now considering approaching a venturecapital organisation for finance.

Identify and discuss five factors that a venture capital organisation would take into account when decidingwhether or not to invest in Cool Ski Co. (10 marks)

(20 marks)

6

Page 56: T10 Question

4 Waste Co is a waste management company, with one sole shareholder/director, Mr Trusty. It collects two types of wastefrom businesses – recyclable waste and confidential waste. Since companies have increasingly become aware of boththe need for recycling and the need to protect confidential information, Waste Co’s client base has expanded rapidlyover the last two years.

As the business has expanded Mr Trusty has had less time available to focus on credit control. This has resulted in asteady deterioration in accounts receivable collection and a rapid increase in Mr Trusty’s overdraft, despite high profits.Mr Trusty’s bank has now refused to extend his overdraft any further and has suggested that he either employ a creditcontroller or factor his accounts receivable.

The following information is available:

1. Credit sales for the year ending 30 November 2007 were $2,550,000 and average accounts receivable dayswere 60. Sales are expected to increase by 25% over the next year.

2. If Mr Trusty employs a good credit controller, the cost to the business will be $47,000. It is anticipated that theaccounts receivable days can then be reduced to 40 days.

3. A local factoring organisation has offered to factor the company’s accounts receivable on the following terms:(i) An advance of 80% of the value sales invoices (which Mr Trusty would fully utilise).(ii) An estimated reduction in accounts receivable days to 35 days.(iii) An annual administration fee of 1·3% of turnover.(iv) Interest charge on advances of 12% per annum.

4. Current overdraft rates are 10% per annum.5. Assume there are 365 days in a year.

Required:

(a) Explain the meaning of ‘debt factoring’ (accounts receivable factoring) to Mr Trusty, distinguishing between‘with recourse’ and ‘without recourse’ agreements. (4 marks)

(b) Explain how debt factoring is different from ‘invoice discounting’. (2 marks)

(c) Calculate whether it is financially beneficial for Waste Co to factor its accounts receivables for the next year,as compared to employing a credit controller. (10 marks)

(d) State four roles that a credit controller may play. (4 marks)

(20 marks)

End of Question Paper

7

Page 57: T10 Question

Managing Finances

ACCA CERTIFIED ACCOUNTING TECHNICIAN EXAMINATION

ADVANCED LEVEL

WEDNESDAY 13 JUNE 2007

QUESTION PAPER

Time allowed 3 hours

ALL FOUR questions are compulsory and MUST be answered

Do not open this paper until instructed by the supervisor

This question paper must not be removed from the examinationhall

The Association of Chartered Certified Accountants

Pape

r T1

0

Page 58: T10 Question

ALL FOUR questions are compulsory and MUST be attempted

1 Go Green Ltd is a small company specialising in the manufacture of a small range of environmentally friendly toiletriesand cleaning products. It is considering extending its production lines to include environmentally friendly soap andwashing-up liquid.

The company’s research and development department has already spent £450,000 developing the products and afurther £325,000 on test marketing them. If the company decides to proceed with the project, new machinery willbe purchased immediately and production will commence straight away.

Notes1. Forecast sales in ‘000 units, are as follows:

Years1 2 3 4 5

Soap 250 200 150 90 30Washing up liquid 240 280 320 170 50

2. Forecasts for the revenues and costs per unit for each of the new products have been made, as detailed below.However, these may need to be revised as per notes (a) to (c) below.

Soap Washing-up liquid£ £

Selling price 1·75 1·60Direct materials (0·15) (0·35)Direct labour (note a) (0·25) (0·12)Variable overheads (note b) (0·50) (0·24)Allocated fixed overheads (note c) (0·65) (0·65)

–––––– ––––––Profit £0·20 £0·24

–––––– –––––––––––– ––––––

(a) The product cost cards for both products above have been prepared on the basis that each factory workeris paid a standard hourly rate of £6. Further details of labour requirements are as follows:

Year oneProduction of each unit of soap requires 2·5 minutes of one factory worker’s time. However, all workers arecurrently working to full capacity. Therefore, for the first year’s production of soap, they will have to workovertime, for which they are paid £7·20 per hour.

Production of each unit of washing-up liquid requires 1·2 minutes of one factory worker’s time. During thefirst year, temporary workers provided by an agency will be used to produce the washing-up liquid at a costto the company of £7 per hour.

Years two to fiveNew permanent staff will be recruited to produce both soap and washing-up liquid. They will be paid at thecompany’s standard hourly rate of £6 per hour.

(b) Variable overheads represent factory power costs, which vary according to labour hours worked.

(c) Allocated fixed overheads comprise factory rental costs of £0·10 per unit and depreciation of £0·55 per unit.

3. The company will need to purchase a new piece of machinery immediately costing £500,000 if the project is toproceed. Some modifications will need to be made to the machinery on site. They will take place as soon as themachinery is purchased and will take two days to complete. The cost of these will be £150,000, payable oncompletion. The machinery will have no scrap value at the end of the project.

4. There are currently two production lines that are not in use on the top floor of the factory. If the project goesahead, these two lines will be used for production. If the project does not go ahead the top floor will be rentedout immediately to a third party, producing income of £125,000 per annum, receivable annually in arrears.

5. Go Green Ltd’s cost of capital is 10% per annum.

6. Assume that all cash flows occur at the end of the year unless otherwise stated.

2

Page 59: T10 Question

7. All workings should be in £’000 to the nearest £’000.

Required:

(a) Using the discount factors provided, calculate the net present value of the proposed project over five yearsand conclude whether the project should be accepted. (20 marks)

(b) Calculate the internal rate of return of the project using the tables provided. (5 marks)

(c) Calculate the discounted payback period for the project at the company’s cost of capital. If the company onlyaccepted projects with a discounted payback period of three years or less, decide whether the companyshould accept this project. (4 marks)

The company has now decided that the project should be funded using debt finance, but it is unsure which form ofdebt finance would be best.

Required:

(d) Discuss three factors that any business should consider when deciding whether a loan or an overdraft is thebest way to finance a project. (6 marks)

(e) Recommend and briefly explain whether an overdraft or a loan would be the most appropriate form of financefor Go Green Ltd. (3 marks)

(f) What is the UK government’s ‘Loan Guarantee Scheme’ and how might it help Go Green Ltd, if required?(2 marks)

Extracts from discount factor tables

Time Factor Factor10% 20%

1 0·909 0·8332 0·826 0·6943 0·751 0·5794 0·683 0·4825 0·621 0·402

(40 marks)

3 [P.T.O.

Page 60: T10 Question

2 Camp Ltd is a retailer of real wood flooring. It buys and sells 20,000 packs of flooring each year from its supplier,Strong Ltd.

The real wood flooring from Strong Ltd costs £35 per pack. There is an order processing charge of £150 per order,irrespective of the quantity of packs ordered, and Strong Ltd takes 10 days to deliver the wood flooring. The averagecost of holding one pack of real wood flooring for one year is £9.

A new supplier of real wood flooring, Fake Ltd, has offered Camp Ltd wood flooring on slightly different terms. FakeLtd guarantees that its wood will never arrive damaged since it uses special packaging designed for maximumprotection. It charges £34·95 per pack for the flooring. There is an order processing charge of £160 per order,irrespective of the quantity of packs ordered, and Fake Ltd takes 7 days to deliver the goods. The average cost ofholding one pack of Fake Ltd’s real wood flooring for one year is £12. This is because the special packaging takes upadditional storage space.

The economic order quantity, which will minimise costs, is:

Where Co = the cost of placing one orderD = the annual demand in unitsCh = the cost of holding one unit per annum.

Required:

(a) Under the current arrangement, calculate:

(i) the economic order quantity for packs of wood flooring;(ii) the total cost (ordering, purchase and holding cost) of stocking wood flooring for one year. (5 marks)

(b) For the new supplier calculate:

(i) the economic order quantity for packs of wood flooring;(ii) the total cost (ordering, purchase and holding cost) of stocking wood flooring for one year (5 marks)

Camp Ltd has decided that it is NOT going to change suppliers. It, therefore, contacts Fake Ltd and informs them thatit does not wish to change suppliers. Fake Ltd has now offered Camp Ltd a bulk purchase discount of 1% on all singleorders for 2,000 or more packs of wood.

Required:

(c) Calculate and conclude whether it is worth accepting the offer by ordering 2,000 packs at a time from FakeLtd. (4 marks)

(d) Outline three non-financial factors that should be taken into account when deciding whether to changesuppliers from Strong Ltd to Fake Ltd. (6 marks)

(20 marks)

4

EOQ CoDCh

= 2

Page 61: T10 Question

3 Porkys Ltd is an ice cream manufacturing company preparing its accounts to 31 May each year. The company’s peakseason occurs in the months of July, August and September and it always prepares a profit forecast for this period.

The following cash budget has been prepared for the six months ending 30 November 2007.

Jun Jul Aug Sep Oct Nov£’000 £’000 £’000 £’000 £’000 £’000

Receipts from sales 1,100 1,250 1,400 1,800 2,200 1,700Sale of machinery – 45 – – – –

–––––– –––––– –––––– –––––– –––––– ––––––Total receipts 1,100 1,295 1,400 1,800 2,200 1,700

–––––– –––––– –––––– –––––– –––––– –––––––––––– –––––– –––––– –––––– –––––– ––––––

PaymentsIngredients 250 280 360 440 340 340Labour 350 450 550 425 275 250Sundry expenses 75 85 95 80 70 65Purchase of machinery – – 120 – 60 –Loan repayments 50 – 50 – 50 –

–––––– –––––– –––––– –––––– –––––– ––––––Total payments 725 815 1,175 945 795 655

–––––– –––––– –––––– –––––– –––––– –––––––––––– –––––– –––––– –––––– –––––– ––––––

Receipts less payments 375 480 225 855 1,405 1,045Opening cash balance b/f 1,400 1,775 2,255 2,480 3,385 4,790Interest received – – – 50 – –

–––––– –––––– –––––– –––––– –––––– ––––––Closing cash balance c/f 1,775 2,255 2,480 3,385 4,790 5,835

–––––– –––––– –––––– –––––– –––––– –––––––––––– –––––– –––––– –––––– –––––– ––––––

Notes/assumptions made when preparing the cash flow forecast1 Customers always take two month’s credit.2 The machinery to be sold for £45,000 will give rise to a profit on disposal of £3,000.3 Depreciation of £13,000 will accrue during the three months ending 30 September 2007.4 One month’s credit is always taken from suppliers of ingredients.5 There are no stocks of raw materials or finished goods maintained.6 Labour is paid by BACS on the last day of each month, for that month’s work.7 Sundry expenses are all paid in the month in which they are incurred.8 The loan repayment relates to an interest free loan obtained from Porkys Ltd’s parent company.9 Interest is received by Porkys Ltd from the bank twice a year. The interest receivable for the peak season of July,

August and September is expected to be £30,000. 10 All workings should be in £’000s to the nearest £’000.

Required:

(a) Prepare a profit forecast for the three months ending 30 September 2007. Show all workings clearly. Whereany items have been excluded from the profit forecast, include a note to justify your treatment of the item.

(10 marks)

Porkys Ltd’s parent company is considering centralising the treasury function for the whole group.

(b) List four roles of a treasury department. (4 marks)

(c) Describe three advantages of having a centralised treasury department. (6 marks)

(20 marks)

5 [P.T.O.

Page 62: T10 Question

4 Jay Ltd manufactures furniture. It has come under pressure in recent years to reduce prices in order to compete withsome larger competitors in the market. The company’s aim has therefore been to maintain sales levels, with the effectthat debtor control has been allowed to deteriorate. The company has always had a target of keeping the debtorsperiod at an average of 45 days.

Debtors as at 31 May 2007 are £323,654. Sales for the year ending 31 May 2007 were £1,581,743. This figureincluded £14,250 of cash sales. During this time, debts of £26,784 were written off. All £26,784 relates to salesmade in the year ending 31 May 2007.

Required:

(a) Calculate the current debtors collection period, in days, from the above information. (3 marks)

(b) How much of the year-end debtors’ balance would have to be immediately recovered in order to reduce thedebt collection period to the target level? (2 marks)

(c) Calculate the company’s bad debt ratio for the year ended 31 May 2007. (2 marks)

(d) List five procedures that could be used to pursue the overdue debts. (5 marks)

Jay Ltd has carried out some of the procedures to collect debts. One of its major debtors is still refusing to pay on thebasis that it was not satisfied with the quality of some of the goods it received and had to make refunds to its owncustomers. Jay Ltd does not want to sue the customer but has heard of ‘arbitration’.

(e) Explain what arbitration is, in this context. (2 marks)

(f) State two advantages and two disadvantages of using arbitration as opposed to taking a case to court.(4 marks)

You have now learnt that arbitration will not be possible since your client informs you that it is actually insolvent. Youhave heard that creditors can attempt to recover monies owing to them through ‘liquidation’.

(g) Briefly explain what happens when a company goes into liquidation. (2 marks)

(20 marks)

End of Question Paper

6

Page 63: T10 Question

Managing Finances

ACCA CERTIFIED ACCOUNTING TECHNICIAN EXAMINATION

ADVANCED LEVEL

WEDNESDAY 13 DECEMBER 2006

QUESTION PAPER

Time allowed 3 hours

ALL FOUR questions are compulsory and MUST be answered

Do not open this paper until instructed by the supervisor

This question paper must not be removed from the examination

hall

The Association of Chartered Certified Accountants

Pape

r T1

0

Page 64: T10 Question

ALL FOUR questions are compulsory and MUST be attempted

1 Morello Landscapes is a small business established several years ago. The owner, Mr Morello, designs and landscapesgardens for a range of clients using his team of five employees. The firm works on one job at a time. Mr Morello hasjust signed a contract with a local building firm to landscape the gardens on a development of thirty executive houses.His designs have already been accepted and he has agreed to complete the work within a six-month period, startingon 1 January 2007. Should he fail to complete the work on time, he will have to pay a penalty. Should the work becompleted early, workers can begin working on the next project.

Mr Morello understands the importance of careful cash budgeting and wants to prepare the cash budget for the nextsix months, from January to June 2007.

The following information has been obtained:

(i) Opening debtors are forecast to be £2,400, all of which will be received in January.

(ii) A price of £400,000 has been agreed for the contract. The amount will be paid in instalments as follows:January 5%February 15%March 10%April 10%May 10%June 50%

(iii) Opening creditors, which will be paid in January, are forecast to be as follows:Materials £6,600Miscellaneous £2,570

(iv) Five diggers will be hired at the start of the job in January to level the land. They will be hired for the wholemonth at a cost of £1,200 each. The fee is payable in full on the first day of hire. A deposit of £500 per diggeris also payable at this point but this amount is refunded in full on the return of the vehicles on the first day ofFebruary.

(v) Various materials are needed to complete the work and these will be purchased at different times over the sixmonths. Ace Ltd supplies all the soil and turf and Hardcastle Ltd supplies the sand, cement and bricks/stones.Shrubs and ‘other materials’ are bought from several different companies. Materials have to be kept on thedriveways of the properties during the landscaping process. Since space is restricted, the following schedule ofpurchases has been drawn up:

Materials Month Purchased Amount Credit termsSoil January £12,600 NoneSand February £2,200 One monthCement February £3,100 One monthBricks/stones March £85,000 Two monthsTurf May £48,000 One monthShrubs May £16,700 NoneOther materials Every month £2,000 per month None

The two key suppliers do charge delivery costs but these are already included in the above amounts. Both keysuppliers also give a 10% bulk order discount on any individual order that exceeds £40,000 in any given month.Mr Morello has not taken any bulk discounts into account when calculating the above figures.

(vi) A waste disposal company has agreed to remove waste throughout the six months at a total cost of £8,500. Thismust be paid in January.

(vii) Each of Mr Morello’s five employees is paid a salary of £21,600 per annum. They are all paid on the last workingday of each month for that month’s work. Mr Morello has also agreed to give each worker a bonus of £1,500 inJune for completion of the contract within the six-month period.

(viii) The firm uses three vans, which the five workers share. These are leased at an annual cost of £3,960 each,payable in equal monthly instalments on the first day of each month.

2

Page 65: T10 Question

(ix) Mr Morello himself uses a business car that has already been fully paid for. He plans to sell the car in April for£3,000 cash, giving rise to an anticipated profit on disposal of £600. His replacement car, to be bought andpaid for in the same month, is expected to cost £18,500. He charges depreciation of £300 each month in hisaccounts for the existing car and will charge £385 per month for the new car. Depreciation is charged in full inthe month of acquisition but not at all in the month of disposal.

(x) Mr Morello’s business account is expected to be overdrawn by £14,200 at the beginning of January.

(xi) The bank charges interest of 1% per month on an overdrawn balance, calculated on the closing bank balanceeach month, and payable the following month. No interest is credited on positive balances.

Required:

(a) Prepare a monthly cash budget for each of the six months to 30 June 2007, showing the cash balance at

the end of each month. Assume that the contract is completed on time. (24 marks)

In the past, Mr Morello has had problems with Hardcastle Ltd delivering the wrong materials or delivering thematerials late. Its prices are so good that he does not want to buy from anybody else. However, it has been such aproblem that he is considering making all of these purchases at the beginning of January. He feels that it would alsobe useful to have a basic understanding of the essential elements of a contract so that he knows his position whendealing with problems with suppliers.

(b) Discuss the costs and benefits, for Mr Morello, of ordering materials early. No additional calculations are

required. (8 marks)

(c) Briefly outline two essential elements of a contract. Do not discuss ‘form’ since most contracts do not have

to be in any strict legal form. (4 marks)

(d) The local building firm that Morello Landscapes has entered into the new contract with has only been in businessfor five years. Mr Morello therefore had to check on the creditworthiness of the firm.

List four external sources of information that Mr Morello may have used to provide assurance about the

creditworthiness of the local building firm. (4 marks)

(40 marks)

3 [P.T.O.

Page 66: T10 Question

2 ‘Nippers’ is a children’s nursery. It is a profitable business and demand for child places always exceeds supply becauseof the great shortage of local nurseries. It is owned and managed by a lady called Mrs Dibble. All the staff at thenursery are either relatives or good friends of Mrs Dibble’s and even the shortest-serving member of staff has workedat the nursery for ten years. Mrs Dibble is planning on retiring on her sixtieth birthday in six years’ time, at whichpoint she hopes to sell the business to one or some of the existing staff. She wants to work full-time until then sinceshe enjoys working so much.

She is currently considering whether to extend the building in order to create more space so that she can meet thedemand for nursery places. Mrs Dibble’s brother has offered to do the extension, by himself, at a very competitiveprice. He is currently unemployed and he faces bankruptcy if he does not find work soon. Mrs Dibble estimates that,with the extension, she would be able to sell the business as a going concern for £600,000 in six years’ time. Withoutthe extension, she would expect to sell it for £500,000 in six years’ time.

A local builder has recently approached Mrs Dibble with an unexpected offer to buy the nursery now for £850,000.He hopes to build apartments on the land.

Mrs Dibble needs to decide whether to carry on in business without the extension (Option 1), have the extension built(Option 2), or sell to the developer (Option 3). The following information is available.

1. Mrs Dibble has already obtained preliminary planning permission for the extension at a cost of £1,200.

2. Mrs Dibble’s building costs are estimated to be £85,000. Of this amount, £45,000 relates to materials and mustbe paid immediately. The balance of the building costs relates to labour and will be paid on completion of thework. The work would take one year to complete. The nursery would still be open as usual during the year sorevenue would be unaffected by the building work.

3. The nursery currently generates net cash inflows of £98,000 per annum. With the extension, these would riseto £135,000 once the work is complete. Mrs Dibble pays herself a salary, but this amount has already beendeducted before arriving at the £98,000.

4. The nursery’s cost of capital is 10% per annum.

5. Assume that all cash flows occur at the end of each year, unless otherwise stated.

Required:

(a) Using the discount tables provided at the end of the question, calculate the net present value (NPV) of each

option at the business’s cost of capital. Based on these calculations, conclude as to which option Mrs Dibble

should choose. (16 marks)

(b) Outline four non-financial factors that Mrs Dibble should take into account before finally making a decision.

(4 marks)

Discount factor tables at a discount rate of 10%Time Factor1 0·9092 0·8263 0·7514 0·6835 0·6216 0·564

Annuity factor tables at a discount rate of 10%Time Factor1 0·9092 1·7363 2·4874 3·1705 3·7916 4·355

(20 marks)

4

Page 67: T10 Question

3 Noise Ltd makes and sells sound equipment to a range of clients. Its main supplier, Speak Ltd, has never offereddiscounts for early payment so Noise Ltd has always taken the full 60 days’ credit allowed by Speak Ltd.

Speak Ltd has recently been having problems collecting its debts on time. Following this, a decision has been madeto offer customers a 2% discount for all invoices paid within two weeks (14 days) of purchase.

Noise Ltd can invest cash to obtain an annual return of 12%.

Required:

(a) Determine whether it is financially viable for Noise Ltd to take advantage of the early payment discount.

(7 marks)

(b) From Speak Ltd’s point of view, what might be three benefits of offering such settlement discounts to its

customers? (3 marks)

(c) Discuss four features of a credit control system that would encourage customers to pay on time.

(10 marks)

(20 marks)

4 Zimmer plc is a listed company specialising in the manufacture and distribution of mobility aids, ranging from walkingsticks to wheelchairs. The company directors are considering branching out into the manufacture and distribution ofstair lifts. Such expansion would require considerable capital investment and the company is therefore consideringhow it could finance the project.

Required:

Explain:

(a) the advantages and disadvantages, to a company, of debt finance over equity finance; (8 marks)

(b) the reasons why a company may choose to issue preference shares rather than ordinary shares or debt;

(4 marks)

(c) four factors that will be taken into account by a bank when deciding whether or not to lend money to a client.

(8 marks)

(20 marks)

End of Question Paper

5

Page 68: T10 Question

Managing Finances

ACCA CERTIFIED ACCOUNTING TECHNICIAN EXAMINATION

ADVANCED LEVEL

WEDNESDAY 14 JUNE 2006

QUESTION PAPER

Time allowed 3 hours

ALL FOUR questions are compulsory and MUST be answered

Do not open this paper until instructed by the supervisor

This question paper must not be removed from the examinationhall

The Association of Chartered Certified Accountants

Pape

r T1

0

Page 69: T10 Question

ALL FOUR questions are compulsory and MUST be attempted

1 Maybay Hospital is a private hospital. Currently, all cleaning is undertaken by staff employed by the hospital. Thereare 150 cleaning staff in total, of which half work full-time and half work part-time.

Over the last year there has been a serious outbreak of the MRSA bug (a bacterial infection resistant to antibiotics) inthe hospital. Cleaning standards have therefore become a key issue. The hospital is currently being sued by a groupof patients who are all claiming to have suffered from MRSA following admittance to Maybay. This has caused asignificant fall in the number of patient admissions and an increase in the hospital’s insurance costs.

The hospital managers are now considering whether they should continue with their existing cleaning staff (Option 1)or contract out the cleaning function to external providers for the next five years (Option 2). The following informationhas been provided:

1 Full-time cleaning staff work 35 hours a week and are paid £5·70 per hour. 2 Part-time cleaning staff work a 20-hour week and are paid £5·65 per hour.3 Eight full-time supervisors are also employed, in addition to the cleaning staff, at a salary of £15,000 per annum

each.4 Other staff costs, including pension costs, average £2,625 per annum per full-time employee (including

supervisors) and £1,215 per part-time employee.5 If the cleaning continues to be done by hospital cleaners, it has already been decided that 10 new full-time

cleaners will be employed immediately. The hospital will also recruit a further 15 full-time cleaners in one year’stime.

6 Total insurance costs for the whole hospital are currently £6·5 million per annum. The hospital estimates thatthese will fall immediately by 10% if the cleaning is contracted out to an external provider. They will remain thesame if the cleaning is NOT contracted out.

7 Cleaning materials cost £1·44 million per annum. If an external provider is used in the future, they will providetheir own cleaning materials.

8 Independent inspectors inspect hospital hygiene standards every six months. Breaches of standards fall into oneof two categories: serious breaches, for which a fine of £10,000 per breach is imposed, and minor breaches, forwhich a fine of £2,000 per breach is imposed. If the hospital does NOT contract out its cleaning, but the newcleaners are employed, it is estimated that serious breaches can be limited to 22 per year and minor breachescan be limited to 74 per year. The external providers expect to restrict these breaches to 17 and 50 respectivelyin the first year, falling to 8 and 25 respectively per year thereafter. Even with a contracted out cleaning service,the hospital will continue to be responsible for all payments of fines.

9 The hospital has invested heavily in floor polishing machines over recent years, and currently has 150 in total.The external providers have offered to buy these immediately at a price of £4,000 each, should the contractingout go ahead.

10 Adverse publicity resulting from the MRSA bug is costing the hospital £1·2 million per annum in lost contribution.The external cleaning providers’ reputation is such that this amount can be totally eliminated immediately.

11 The external providers have offered to provide cleaning services for a contracted five-year period. The fees will be£4·25 million for each of the first two years, increasing to £4·5 million per annum thereafter.

12 Administration costs are currently £300,000 per annum. They would fall to £270,000 per annum if the cleaningwere contracted out.

13 If the hospital decides to contract out the cleaning, it will be with immediate effect. A redundancy package hasbeen put together for current staff. Each full-time employee, including supervisors, will receive an average of£5,000 and each part-time employee will receive an average of £3,000. These amounts will be payableimmediately.

14 The hospital managers have spent £60,000 researching and calculating their costs.15 The hospital’s cost of capital is 10% per annum.16 Assume that all cash flows occur at the end of each year unless told otherwise.

2

Page 70: T10 Question

Required:

(a) Calculate the net present values of the costs of each of the two options above. Recommend whether thehospital should continue with its existing staff (Option 1) or contract out of cleaning for the next five years(Option 2). (30 marks)

(b) State four factors, other than cost, that should be taken into account when deciding whether to use theexternal providers. (4 marks)

(c) Explain the main principles used to differentiate between relevant and irrelevant costs for investmentappraisal. Include a brief explanation of the treatment of finance costs. (6 marks)

NOTE: All workings and answers should be in £’000, to the nearest £’000.

Present value table (extract)Periods (n) Discount rate (r)

10%1 0·9092 0·8263 0·7514 0·6835 0·621

Annuity factor table (extract)Periods (n) Discount rate (r)

10%1 0·9092 1·7363 2·4874 3·1705 3·791

(40 marks)

3 [P.T.O.

Page 71: T10 Question

2 You have been provided with the following financial information relating to Health Foods Ltd.

Forecast Profit and Loss Account for the year ending 30 June 2007

£’000Turnover 20,350Operating costs (12,265)

–––––––Operating profit 8,085Interest payable (785)

–––––––Profit before tax 7,300Tax payable (2,230)

–––––––Profit after tax 5,070Dividends payable (2,270)

–––––––Retained profit 2,800

–––––––

Extract from Historical Balance Sheet as at 30 June 2006 (Actual figures)

£’000 £’000 £’000Fixed assets 8,000Current assetsStock 2,167Debtors 2,543Cash 1,264

––––––5,974

Current liabilitiesTrade creditors 1,737Tax payable 1,895Dividends payable 1,542

––––––(5,174)––––––

Net current assets 800––––––

Net assets 8,800––––––––––––

Extract from Forecast Balance Sheet as at 30 June 2007

£’000 £’000 £’000Fixed assets 8,300Current assetsStock 3,245Debtors 3,318Cash 2,984

––––––9,547

Current liabilitiesTrade creditors 1,723Tax payable 2,289Dividends payable 2,235

––––––(6,247)––––––

Net current assets 3,300–––––––

Net assets 11,600––––––––––––––

4

Page 72: T10 Question

Additional information1. Operating costs include depreciation of £200,000.2. The company does not plan to sell any fixed assets over the next year.3. Fixed assets are all tangible (physical) assets.4. All finance costs are paid in the year in which they are incurred.

Required:

(a) Prepare a forecast cash flow statement for Health Foods Ltd for the year ending 30 June 2007 identifyingthe net cash flow for the business.Note: Accounting standards format is not required. (16 marks)

(b) Briefly describe the drawbacks of the Baumol (EOQ) cash management model. (4 marks)

(20 marks)

5 [P.T.O.

Page 73: T10 Question

3 You are an accounting technician working at Shoes for You Ltd, a company that manufactures and distributes a rangeof fashion shoes. All shoes are made at the company’s factory in the country of Bushai, where materials and labourhave historically been very cheap. The shoes are then exported to the UK where they are sold to a number of retailoutlets.

All costs are incurred in Bushai’s unit of currency. All materials are paid for in cash at the time of purchase. Productionstaff are paid their wages daily in cash. They have not had a pay increase in the last year. All other overheads,production and sales, are on credit.

All sales are to UK customers and are on credit. They are, therefore, invoiced (and amounts are received) in £ sterling.Any finance needed for the business is also obtained from the UK.

You have estimated the figures below for the coming year. Sterling has been used for all figures so as to avoid anydistortion caused by high inflation in Bushai.

Sales £2,500,000Average debtors £410,000Materials purchases £630,000Production staff wages £450,000Other production overheads £350,000Sales overheads £320,000Net profit margin 30%Average stocks:Finished goods £325,000Work in progress (65% complete) £195,000Raw materials £133,000Average creditors £73,000

The economy in Bushai has recently become unstable. This has led to a rapid increase in inflation levels over the lastyear, from 10% at the beginning of the year to 25% at the end of the year. Interest rates are controlled by Bushai’scentral bank. Inflation in the UK has remained stable at about 4% per annum.

Required:

(a) Calculate the cash operating cycle. (10 marks)

(b) The raw material holding period has doubled over the last year.

State the main financial advantage and main financial disadvantage of this happening, paying attention tothe fact that inflation has increased over the year. (4 marks)

(c) Discuss BRIEFLY the general problems associated with inflation as listed below and consider how eachproblem affects Shoes for You Ltd.

(i) Redistribution of income and wealth; (2 marks)

(ii) The balance of trade (Do NOT discuss exchange rates); (2 marks)

(iii) Higher interest rates. (2 marks)

(20 marks)

6

Page 74: T10 Question

4 Financial analysts will use ratios to compare performance of companies in the same industry.

Lenders will frequently use ratio analysis to help them decide whether to lend to an individual in the first place andwhether to continue their financial support. Business owners and managers also use ratios to assess the financialperformance of their business. Such ratios may include earnings per share, interest cover, gearing and net profitmargin.

Required:

(a) Outline FOUR sources of finance (short and/or long-term) available to small and medium-sized businesses.Ignore government grants, leasing and factoring. (12 marks)

(b) Discuss four limitations of ratio analysis. (8 marks)

(20 marks)

End of Question Paper

7

Page 75: T10 Question

Managing Finances

ACCA CERTIFIED ACCOUNTING TECHNICIAN EXAMINATION

ADVANCED LEVEL

WEDNESDAY 14 DECEMBER 2005

QUESTION PAPER

Time allowed 3 hours

ALL FOUR questions are compulsory and MUST be answered

Do not open this paper until instructed by the supervisor

This question paper must not be removed from the examinationhall

The Association of Chartered Certified Accountants

Pape

r T1

0

Page 76: T10 Question

ALL FOUR questions are compulsory and MUST be attempted

1 Gold Club Ltd is a newly established company that has not yet commenced trading. The two shareholders/directorsare planning to set up an Internet business. With the help of finance from venture capitalists, they plan to set up awebsite through which individuals can join a ‘Gold Club’. Membership of the club will provide members with a rangeof quality travel services including access to a First Class lounge at airports, travel insurance for an unlimited numberof trips and discounted upgrades for seats on flights. It will take one year to set up the business.

You are an accounting technician who has been asked to appraise the investment of capital in the project based ona six-year plan (one year to set up the business followed by five years of operation).

(i) Market research has been carried out at a cost of £200,000. This indicates that the number of new membersjoining the club would be approximately 5,000 each year. Of these new joiners each year, 50% will renew theirmembership ONCE, in the following year.

(ii) Membership fees for new members joining the club each year will be £500. Members renewing theirmembership in the subsequent year will benefit from a reduced annual fee of £400.

(iii) The costs of setting up the website are as follows:– Design costs: £1,220,000– Administrative costs: £125,000– Legal advice fees: £155,000

25% of these costs will be paid immediately as a deposit, with the remainder being paid in one year’s time.

(iv) The costs of the technical support for maintaining the website will be £100,000 for the first year it is in operation.These costs will increase by 20% year-on-year thereafter.

(v) All new members will be issued with a club card, with an electronic chip in it. The cost of this card for Gold ClubLtd will be £3 per member. Once the original card is issued, there will be no need to issue a further card forrenewal of membership. The supplier will be paid annually in arrears for all cards issued in that year. The firstpayment is therefore due at the end of the second year.

(vi) For every member who uses a First Class lounge at any airport worldwide, a charge of £20 will be made to GoldClub Ltd each time the lounge is used. It is estimated that 40% of members will use it three times a year, 30%of members will use it twice a year, 20% will use it once a year and 10% will not use it at all.

(vii) Gold Club Ltd will purchase a travel insurance policy based primarily on the number of members in the scheme.It is assumed that all members will make use of the travel insurance and the supplier will therefore charge GoldClub Ltd £50 per member in the first year, increasing steadily by £2 per member each year after that. Eachmember will have to e-mail the travel insurers with details of their trip in advance of each trip. Therefore, theinsurers will also charge Gold Club Ltd an administrative fee of £2 for every e-mail received. Gold Club Ltdestimates that on average each member will send two e-mails per annum.

(viii) Gold Club Ltd will pay ten major airlines £100,000 EACH per annum in order to provide their members withaccess to discounted seat upgrades.

(ix) Overdraft interest will be £5,000 per annum.

(x) Assume that all cash flows, including the receipt of membership fees, occur at the end of each year, unlessotherwise stated.

(xi) The company’s cost of capital is 10% per annum.

(xii) Workings should be in £’000, to the nearest £’000.

2

Page 77: T10 Question

Required:

(a) Using the discount tables provided at the end of the question, calculate the project’s net present value (NPV)at the company’s cost of capital. Conclude as to whether the company should proceed with the project,giving a reason for your conclusion. (30 marks)

(b) Identify and discuss five factors that a venture capital organisation would take into account when decidingwhether or not to invest in Gold Club Ltd. (10 marks)

Present value table (extract)Periods (n) Discount rates (r)

10% 20%1 0·909 0·8332 0·826 0·6943 0·751 0·5794 0·683 0·4825 0·621 0·4026 0·564 0·3357 0·513 0·2798 0·467 0·2339 0·424 0·19410 0·386 0·162

(40 marks)

3 [P.T.O.

Page 78: T10 Question

2 Cleanly Ltd is a manufacturing company producing and selling a range of cleaning products to wholesale customers.It has three suppliers and two customers. Cleanly Ltd relies on its cleared funds forecast to manage its cash.

You are an accounting technician for the company and have been asked to prepare a cleared funds forecast for theperiod Monday 2 January to Friday 6 January 2006 inclusive. You have been provided with the following information:

(1) Receipts from customersCustomer name Credit Payment 2 Jan 2006 2 Dec 2005 sales

terms method salesW Ltd 1 calendar month BACS £150,000 £130,000X Ltd None Cheque £180,000 £160,000

(a) Receipt of money by BACS is instantaneous.

(b) X Ltd’s cheque will be paid into Cleanly Ltd’s bank account on the same day as the sale is made and willclear on the third day following this (excluding day of payment).

(2) Payments to suppliersSupplier Credit Payment 2 Jan 2006 2 Dec 2005 2 Nov 2005name terms method purchases purchases purchasesA Ltd 1 calendar month Standing order £65,000 £55,000 £45,000B Ltd 2 calendar months Cheque £85,000 £80,000 £75,000C Ltd None Cheque £95,000 £90,000 £85,000

(a) Cleanly Ltd has set up a standing order for £45,000 a month to pay for supplies from A Ltd. This will leaveCleanly’s bank account on 2 January. Every few months, an adjustment is made to reflect the actual costof supplies purchased (you do NOT need to make this adjustment).

(b) Cleanly Ltd will send out, by post, cheques to B Ltd and C Ltd on 2 January. The amounts will leave itsbank account on the second day following this (excluding the day of posting).

(3) Wages and salariesDecember 2005 January 2006

Weekly wages £12,000 £13,000Monthly salaries £56,000 £59,000

(a) Factory workers are paid cash wages (weekly). They will be paid one week’s wages, on 6 January, for thelast week’s work done in December (i.e. they work a week in hand).

(b) All the office workers are paid salaries (monthly) by BACS. Salaries for December will be paid on 2 January.

(4) Other miscellaneous payments(a) Every Monday morning, the petty cashier withdraws £200 from the company bank account for the petty

cash tin. The money leaves Cleanly’s bank account straight away.

(b) The window cleaner is paid £30 from petty cash every Wednesday morning.

(c) Office stationery will be ordered by telephone on Tuesday 3 January to the value of £300. This is paid forby company debit card. Such payments are generally seen to leave the company account on the nextworking day.

(d) Five new computers will be ordered over the Internet on 5 January at a total cost of £6,500. A cheque willbe sent out on the same day. The amount will leave Cleanly Ltd’s bank account on the second day followingthis (excluding the day of posting).

(5) Other informationThe balance on Cleanly’s bank account will be £200,000 on 2 January 2006. This represents both the bookbalance and the cleared funds.

Required:

Prepare a cleared funds forecast for the period Monday 2 January to Friday 6 January 2006 inclusive using theinformation provided. Show clearly the uncleared funds float each day.

(20 marks)

4

Page 79: T10 Question

3 Fibre Clean Ltd is a small company specialising in the cleaning of upholstery and carpets. All of the company’s currentcustomers are domestic and pay at the time the work is done. Mr Sykes, the owner, is considering undertaking anumber of commercial contracts instead of his domestic work because the profit margins are much higher. It isstandard practice in this sector to offer 30-day credit terms to customers. Mr Sykes is concerned about a number ofissues. Firstly, how he can ensure that he only provides credit to creditworthy customers. Secondly, how he willmanage the extra administrative workload and, finally, what the effect of providing credit will be on his cash position.He is already overdrawn at his local bank.

A friend of Mr Sykes has suggested that he should obtain ‘trade references’ for his new customers and seek the adviceof a ‘credit reference agency’.

Required:

(a) Describe the following sources of externally generated information and their usefulness in assessing thecreditworthiness of Mr Sykes’ new customers:

(i) Trade reference;(ii) Credit reference agency. (6 marks)

(b) Explain the meaning of ‘debt factoring’ and ‘invoice discounting’ to Mr Sykes. Highlight any key differencesbetween the two and suggest which one may be more useful for Mr Sykes. (5 marks)

(c) If Mr Sykes changes to commercial contracts, he expects sales for the next year to be £75,000, with customerspaying within the 30-day limit set. It would cost him £2,000 per annum to employ someone one day a week toinvoice customers and collect debts for him. Alternatively, his local bank has offered to provide a factoring servicefor him, including the advance of 80% of his sales invoices. They would charge 2% of turnover for theadministration and charge interest of 8% per annum on advances. However, the bank would not invoice Mr Sykes’ customers. He would need to employ somebody for half a day a week to do this, at a cost of £1,000per annum.

Mr Sykes pays interest at the rate of 10% per annum on his overdrawn bank account.

Mr Sykes thinks that customers will pay within 30 days regardless of which option is selected.

Required:

Calculate and recommend whether or not Mr Sykes should factor his debts. (9 marks)

(20 marks)

5 [P.T.O.

Page 80: T10 Question

4 (a) Define the term ‘financial intermediary’, distinguishing between a ‘broker’ and a ‘principal’. Give fourexamples of organisations that act as financial intermediaries. (4 marks)

(b) Discuss four main benefits of financial intermediation. (8 marks)

(c) Briefly describe four main money market instruments. (4 marks)

(d) Governments use monetary policy to control prices, economic growth and employment levels.

Required:

Define ‘quantitative’ and ‘qualitative’ controls used by governments. (4 marks)

(20 marks)

End of Question Paper

6

Page 81: T10 Question

Managing Finances

ACCA CERTIFIED ACCOUNTING TECHNICIAN EXAMINATION

ADVANCED LEVEL

WEDNESDAY 15 JUNE 2005

QUESTION PAPER

Time allowed 3 hours

ALL FOUR questions are compulsory and MUST be answered

Do not open this paper until instructed by the supervisor

This question paper must not be removed from the examinationhall

The Association of Chartered Certified Accountants

Pape

r T1

0

Page 82: T10 Question

ALL FOUR questions are compulsory and MUST be attempted

1 Seventeen LtdSally Sharp is the Managing Director of Seventeen Ltd, a production company responsible for the huge success of thetelevision programme ‘Pop Icon 1’ and ‘Pop Icon 2.’ The programme is just nearing the end of the second series, withonly four weeks to go before the public chooses a winner.

Sally is in the process of deciding whether to run a ‘Pop Icon’ tour. This would consist of twelve live performances, intwelve different locations, during the month of September. The twelve finalists from the television programme wouldall take part in the tour. Seventeen Ltd ran a ‘Pop Icon 1’ tour after the first series and this was a big success. Sallyis concerned, however, about the effect of the tour on the company’s cash flow, as the company currently has fundstied up in several other projects.

You have been provided with the following information:

(i) Seventeen Ltd will open a separate bank account on 1 July 2005 for the tour, with an opening balance of£500,000.

(ii) During the Pop Icon 1 tour, it was calculated that for each thousand votes cast during the final twelve televisionprogrammes, one person attended the Pop Icon tour. This ratio is expected to remain the same for the secondPop Icon tour, should it go ahead.

During the last eight weeks of Pop Icon 2, the public has cast a total of 180 million votes. This is an average of22·5 million votes per week. Public interest is expected to increase over the final four weeks and it is thereforeestimated that votes for the remaining four weeks will be 100 million in total.

(iii) Tickets vary in price according to the seat location within each venue. Ticket information is as follows:

Ticket type Price Percentage of total tickets soldPremium seats £35 10%Arena floor £30 30%Stalls £25 60%

It is expected that 20% of ticket revenues will be received in September, 30% in October, 30% in November andthe remainder in December.

(iv) At every live performance, a selection of souvenirs will be on sale, from mugs and t-shirts to CDs and DVDs.From the Pop Icon 1 tour it has been calculated that for every ticket sold, an average of £6 is spent on souvenirs.These monies are received by Seventeen Ltd in September.

(v) The souvenirs are purchased from two key suppliers – Records Ltd, who produce all the CDs and DVDs (50% oftotal souvenirs sales), and Junk Ltd who produce everything else (remaining 50% of sales). The gross profitmargins for Seventeen Ltd are 25% for CDs and DVDs and 50% for everything else. All orders must be madefrom both suppliers in August but whereas Records Ltd requires payment at the time of order, Junk Ltd providesone month’s credit.

(vi) Each of the twelve performers will be paid £35,000 for his/her participation in the live tour. Of this amount,£5,000 will be paid at the start of the tour at the beginning of September, with the remainder being paid at thebeginning of October.

(vii) Seventeen Ltd will pay an average of £150,000 to hire each of the twelve venues for the live tour. In order tosecure the venues, a deposit of 10% had to be paid in May for each venue out of one of the company’s existingbank accounts. The remaining balances will be paid in August from the Tour bank account.

(viii) Seventeen Ltd will need to hire two tour buses in September – one for the performers and one for the crew. Thecost of these will be £10,000 each. In addition to these, six heavy vehicles will be required to transportequipment. Each vehicle costs £12,000. All of these transport costs include driver costs and fuel. 75% of thetotal transport costs (including buses) will be paid in September, with the remainder being paid in October.

(ix) For each performer on the tour, five members of crew are required over and above the already existing staff ofSeventeen Ltd. The average gross wage of each crewmember is £12,000 per month. Whilst all of thesecrewmembers are required for the month of September, only half of them are needed for August. Allcrewmembers are paid at the end of each month that they work.

2

Page 83: T10 Question

(x) The crewmembers are hired through an agency, Hands on Deck Ltd. The agency charges a fee for each memberof crew hired through them. The fee is calculated as 10% of the gross monthly wage of each crewmember andis paid for EVERY month that they work. Hands on Deck Ltd allows one month’s credit.

(xi) Publicity for the tour is expected to cost a further £2·5 million. Of this amount, 20% will be paid in July and30% paid in August. Sally has negotiated delaying the remaining payment until November.

(xii) Other costs are estimated to be in the region of £240,000 IN TOTAL, incurred evenly through the months of July,August, September and October. They will be paid in the month in which they are incurred.

(xiii) All workings should be in £’000, to the nearest £’000.

Required:

(a) Prepare a monthly cash budget for the Pop Icon 2 Tour for EACH of the six months to 31 December 2005,showing clearly any necessary workings. (30 marks)

The following information applies to part (b) only.Sally is concerned that the above calculations rely on the public’s average weekly votes increasing in the final fourweeks of the programme. They also rely on the assumption that there will be one ticket sale per thousand votes. Sallyis concerned about the effect on profit should these assumptions be wrong, with the second tour proving to be lesspopular that the first one. She now wants you to assume that voting for the final four weeks of the programme staysat the same average level as the previous eight weeks AND that for every TWO thousand votes only one person attendsthe tour.

(b) Calculate the original profit of the project as per part (a) AND the revised profit/loss after the change in theassumptions. (10 marks)

(40 marks)

3 [P.T.O.

Page 84: T10 Question

2 Silly Filly LtdSilly Filly Ltd is a recently established company specialising in the manufacture of talking toy horses for children. TheSilly Filly range currently comprises three key products – all of which are toy horses – plus approximately thirtyaccessories to complement the range, from stables to grooming kits.

The Silly Filly range has been such a success in the last year that the management is considering producing ananimated film to accompany the range. This is in accordance with the company’s long-term expansion plans,culminating in a stock exchange flotation in three year’s time.

The film will take one year to make. In the year following that, sales of the film will commence.

You, an accounting technician for the company, have been asked to assist in appraising the project to decide whetherit should go ahead. The following information is relevant to your calculations.

(i) Market research has already been carried out at a cost of £1·2 million.

(ii) The services of a company specialising in animation will be required at a total cost of £520,000. 50% of thesecosts will be paid immediately with the remainder being paid in one year’s time.

(iii) Two producers will be employed throughout the first year of the project. They will each be paid salaries of£120,000.

(iv) Other production costs during the year are expected to be £650,000.

(v) A film director will be employed immediately on a one-year contract at a cost of £160,000.

(vi) The animated film is expected to generate revenues of £1·2 million in the first year of sales, £2·2 million in thesecond year, and £1·6 million in the third year.

(vii) The two producers and the director will each be paid royalties from the film. These will be paid at the rate of1·5% of gross revenues for EACH of the producers and 2% for the director. They will always be payable one yearin arrears.

(viii) Specialist equipment will need to be purchased immediately for the film production. This will cost £2·3 millionbut can be sold at the end of the year for £1·7 million.

(ix) A loan for £1 million will be taken out to assist in financing the project. The loan will be repayable in two year’stime, with interest of 8% per annum being payable for its duration.

(x) The company’s cost of capital is 10% per annum.

(xi) Assume that all cash flows occur at the end of each year, unless otherwise stated.

Required:

(a) Using the present value tables provided at the end of the question, calculate the project’s net present value(NPV) at the company’s cost of capital. Conclude as to whether the company should proceed with theproject, giving a reason for your conclusion. (10 marks)

(b) List four costs associated with a new equity issue. (4 marks)

(c) Explain three reasons why a company may seek a stock exchange listing. (6 marks)

Present Value table (extract)Periods (n) Discount rate (r) Discount rate (r)

10% 8%1 0·909 0·9262 0·826 0·8573 0·751 0·7944 0·683 0·7355 0·621 0·681

(20 marks)

4

Page 85: T10 Question

3 Duel Fuel LtdDuel Fuel is an energy company that buys gas in bulk and sells it to the main market providers in the UK. Thecompany’s credit control department is poorly run at present, following the resignation six months ago of the creditcontroller. Poor credit control has had a serious effect on the company’s overdraft, which currently stands at £5million, compared to £3 million at the same time last year. This, in turn, is pushing up interest costs.

The company is therefore considering factoring its debts to improve its cash flow and reduce costs.

Credit sales for the last year totalled £8·5 million, with average debtors of £4·3 million. Next year, sales are expectedto be 20% higher, and debtor days are expected to remain the same if the factoring arrangement is NOT entered into.

A factoring company has put forward the following proposal to Duel Fuel Ltd:

(i) The factor would immediately advance 70% of the value of sales invoices.(ii) The factor will charge interest of 12% per annum on the advances.(iii) The factor will charge an administration fee of 1·5% of turnover for the service.(iv) Debtor days will be reduced to 60 days, the industry norm, as a result of stricter credit control procedures

Should Duel Fuel Ltd enter into the agreement, it will make both of the credit control staff redundant. They earnsalaries of £14,000 per annum each, but they will both be paid a redundancy package of £3,000 each.

Current bank overdraft rates are 9% per annum.

Required:

(a) Calculate for the coming year whether it is financially viable for Duel Fuel Ltd to factor its debts.(10 marks)

(b) Identify five advantages of factoring. (5 marks)

(c) List five functions that a credit control department may undertake. (5 marks)

(20 marks)

5 [P.T.O.

Page 86: T10 Question

4 Gym Jam LtdGym Jam Ltd is a well-established company that hires out a range of top quality treadmills to gyms across the country,under operating leases.

Over recent months, Gym Jam Ltd’s client base has expanded so rapidly that the company has struggled to financethe level of treadmill purchases required to supply its new clients. The company used a loan from its local bank toassist with its last bulk order of treadmill purchases two years ago. Interest on this loan was variable, being linked tothe bank’s base rate, and Gym Jam Ltd saw its interest charges steadily increasing over the two-year period. It isnow considering using finance leases (fixed interest) to acquire the next bulk order of treadmills.

In the last year, the company’s debtor days have increased from an average of 30 days to 45 days, partly becausesales ledger staff could not cope with the increased workload. Currently, all customers are invoiced every three months(quarterly) for the hire of the treadmills, and most of them pay by cheque. Invoices are raised manually and state onthem that payment terms are 30 days from invoice date.

You are an accounting technician for the company, and have been asked to assist in resolving these problems.

Required:

(a) Briefly describe three key characteristics of an operating lease. (3 marks)

(b) Briefly describe five key characteristics of a finance lease. (5 marks)

(c) Identify four weaknesses in Gym Jam Ltd’s invoicing and credit control system and indicate how they canbe rectified. (4 marks)

(d) List the factors that affect the rate of interest for a loan. (6 marks)

(e) Explain why the interest charges on Gym Jam Ltd’s loan kept increasing. (2 marks)

(20 marks)

End of Question Paper

6

Page 87: T10 Question

Managing Finances

ACCA CERTIFIED ACCOUNTING TECHNICIAN EXAMINATION

ADVANCED LEVEL

WEDNESDAY 15 DECEMBER 2004

QUESTION PAPER

Time allowed 3 hours

ALL FOUR questions are compulsory and MUST be answered

Do not open this paper until instructed by the supervisor

This question paper must not be removed from the examinationhall

The Association of Chartered Certified Accountants

Pape

r T1

0

Page 88: T10 Question

ALL FOUR questions are compulsory and MUST be attempted

1 Tots Ltd specialises in the importation and sale of equipment for children’s indoor play centres. The company was setup two years ago by its joint shareholders, Mr and Mrs Brute.

The business has been very successful, expanding rapidly over the last year, and the cash balance in the company’scurrent account has exceeded £1 million on several occasions recently.

Mr and Mrs Brute have asked you, an accounting technician for Tots Ltd, to assist them in managing their cashbalances over the next six months.

You have been provided with the following information.

(i) The bank balance on 1 January 2005 is forecast at £1·2 million in credit.(ii) Sales for November and December 2004 are £1·3 million per month. They are expected to rise to £1·5 million

in January 2005, £1·7 million in February and £1·9 million in March. They will then fall to £1·4 million foreach of the following six months. This is due to a downturn in demand as the weather improves.

(iii) All sales are made on credit. 2% of debtors do not pay at all, 70% pay one month after sale and the remaining28% pay two months after sale.

(iv) Purchases are made one month prior to sales, and two months’ credit is taken from suppliers. (v) The company’s gross profit margin is 50%.(vi) The cost of employing Tots Ltd’s permanent staff is £150,000 per month. Tots Ltd also employs temporary staff

during January, February and March at an additional cost equating to 3% of sales each month.(vii) Tots Ltd uses a courier to despatch the equipment to its customers. The cost of this service is 2% of sales value

in January to March, falling to 1% thereafter. (viii) Administration costs are forecast at £30,000 for January. These costs are directly proportional to sales each

month.(ix) Mr and Mrs Brute will be attending a conference abroad in July 2005 at a total cost of £5,000. They must

complete the booking form and send it off, along with a deposit of £2,000, by the end of January 2005. Thefinal balance is due in June.

(x) The company charges depreciation of £45,000 each month.(xi) Tots Ltd also owns two indoor play centres that it rents out at the rate of £3,500 each per month from January

to April, falling to £3,000 per month thereafter. All rents are received one month in advance.(xii) The company will invest in a new computer system later in the year. This will be paid for by two equal

instalments of £200,000, one in June and one in September.

Required:

(a) Prepare a monthly cash budget for EACH of the six months to 30 June 2005, showing clearly any necessaryworkings.

NOTE: All workings should be in £000.Unless told otherwise, assume that payments are made in the month in which the costs are incurred.

(16 marks)

Mr and Mrs Brute are aware that the business is holding too much cash, but are unsure how to invest it safely. Theyare very risk averse, having each lost a considerable amount of money on the Stock Exchange. They also want toensure that they retain enough cash to allow the business to meet its debts as they fall due.

(b) Briefly explain three motives for holding cash, as identified by Keynes. (6 marks)

2

Page 89: T10 Question

(c) Define and explain the characteristics of THREE out of the four types of investment below. In light of thecharacteristics you have identified, conclude as to whether you consider each of the investments you haveexplained to be an appropriate use of Tots Ltd’s cash surplus.

(i) Certificates of deposit;(ii) Gilt-edged securities (gilts);(iii) Shares;(iv) Bills of exchange. (18 marks)

Note: Each type of investment carries equal marks.

(40 marks)

3 [P.T.O.

Page 90: T10 Question

2 Pooch Ltd makes and sells pet care products. The following projected data for the next year is available.

Sales £2,200,000

Costs as percentage of salesRaw materials 15%Direct labour 20%Variable production overheads 11%Fixed production overheads 10%Other costs 12%

Working capital statisticsAverage raw material holding period 4 weeksAverage Work-in-progress (WIP) holding period 2 weeksAverage finished goods holding period 4 weeksAverage debtors’ collection period 6 weeksAverage creditors’ payment period on:Raw materials 4 weeksDirect labour 1 weekVariable production overheads 8 weeksFixed production overheads 5 weeksOther costs 12 weeks

Other relevant information– All finished goods stock and WIP values include raw materials, direct labour, variable production overheads and

apportioned fixed production overhead costs.

– Assume WIP is 75% complete as to materials and 50% complete as to direct labour, variable productionoverheads and fixed production overheads.

– Assume there are 52 weeks in one year.

– Assume that production and sales volumes are the same.

Required:

(a) Calculate the estimated average working capital required by Pooch Ltd for the year, showing clearly allnecessary workings.

NOTE: All workings should be in £.(14 marks)

(b) Pooch Ltd is currently in dispute with one of its suppliers, Petcoats Ltd. Pooch Ltd is unsure of its legalobligations regarding an agreement it made with Petcoats Ltd. The Financial Director of Pooch Ltd has askedyou, an accounting technician, to explain certain legal terms to him.

Define and briefly explain the following legal terms:

(i) ‘Offer’; (2 marks)

(ii) ‘Acceptance’; (2 marks)

(iii) ‘Consideration’. (2 marks)

(20 marks)

4

Page 91: T10 Question

This is a blank pageQuestion 3 begins on page 6

5 [P.T.O.

Page 92: T10 Question

3 Taxi Ltd is a long established company providing high quality transport for customers. It currently owns and runs 350cars and has a turnover of £10 million per annum.

The current system for allocating jobs to drivers is very inefficient. Taxi Ltd is considering the implementation of a newcomputerised tracking system called ‘Kwictrac’. This will make the allocation of jobs far more efficient.

You are an accounting technician for an accounting firm advising Taxi Ltd. You have been asked to perform somecalculations to help Taxi Ltd decide whether Kwictrac should be implemented. The project is being appraised over fiveyears.

The costs and benefits of the new system are set out below.

(i) The central tracking system costs £2,100,000 to implement. This amount will be payable in three equalinstalments: one immediately, the second in one year’s time, and the third in two years’ time.

(ii) Depreciation on the new system will be provided at £420,000 per annum.

(iii) Staff will need to be trained how to use the new system. This will cost Taxi Ltd £425,000 in the first year.

(iv) If Kwictrac is implemented, revenues will rise to an estimated £11 million this year, thereafter increasing by 5%per annum (i.e. compounded). Even if Kwictrac is not implemented, revenues will increase by an estimated£200,000 per annum, from their current level of £10 million per annum.

(v) Despite increased revenues, Kwictrac will still make overall savings in terms of vehicle running costs. These costsavings are estimated at 1% of the post Kwictrac revenues each year (i.e. the £11 million revenue, rising by 5%thereafter, as referred to in note (iv)).

(vi) Six new staff operatives will be recruited to manage the Kwictrac system. Their wages will cost the company£120,000 per annum in the first year, £200,000 in the second year, thereafter increasing by 5% per annum(i.e. compounded).

(vii) Taxi Ltd will have to take out a maintenance contract for the Kwictrac system. This will cost £75,000 per annum.

(viii) Interest on money borrowed to finance the project will cost £150,000 per annum.

(ix) Taxi Ltd’s cost of capital is 10% per annum.

Required:

(a) Calculate the net present value of the new Kwictrac project to the nearest £000. Use the discount factorsprovided at the end of the question. (10 marks)

(b) Calculate the simple payback period for the project and interpret the result. (3 marks)

(c) Calculate the discounted payback period for the project and interpret the result. (3 marks)

6

Page 93: T10 Question

(d) Taxi Ltd wants to ensure that it has enough cash available to pay the second and third instalments for theKwictrac system, when they fall due. The company has therefore decided to invest the cash on time depositswith its local bank. The rates of interest paid by the bank are as follows:

6 month deposits 7% per annumOne year deposits 8% per annumTwo year deposits 9% per annumThree year deposits 10% per annum

Interest is paid once a year, at the end of the year.

Calculate the total amount of cash that Taxi Ltd needs to put on deposit immediately in order to meet thefinal two instalments for Kwictrac. (4 marks)

NOTE: You should assume that all cash flows occur at the end of the year, unless otherwise stated.

Present Value table (extract)

Periods (n) Discount rate (r)10%

1 0·9092 0·8263 0·7514 0·6835 0·621

(20 marks)

4 Skint Ltd is a small family owned company that makes fuses for electrical plugs. It was set up twenty-five years agoby its main shareholder, Mr Holmes, who is also the Managing Director of the company.

The company is facing short-term cash flow difficulties. It is already a highly geared company and Mr Holmes isconcerned that the bank will not lend it any more money. He is considering applying for a personal loan or giving apersonal guarantee in order to solve the company’s short-term cash flow difficulties.

Required:

(a) List and explain the general factors that will be taken into account by a bank when deciding whether or notto lend money to a client. (14 marks)

(b) Briefly explain three characteristics that any security for a loan should have. (3 marks)

(c) Briefly describe the following:

(i) bullet repayment loan;(ii) balloon repayment loan;(iii) amortising loan (straight repayment loan). (3 marks)

(20 marks)

End of Question Paper

7

Page 94: T10 Question

Managing Finances

ACCA CERTIFIED ACCOUNTING TECHNICIAN EXAMINATION

ADVANCED LEVEL

WEDNESDAY 16 JUNE 2004

QUESTION PAPER

Time allowed 3 hours

ALL FOUR questions are compulsory and MUST be answered Pape

r T1

0

The Association of Chartered Certified Accountants

Page 95: T10 Question

ALL FOUR questions are compulsory and MUST be attempted

1 Paradise Ltd is a large company specialising in luxury holidays for the rich and famous. It has recently purchased anuninhabited island, close to the popular resort of Luca, at a cost of £2 million. The company has already spent £1·5 million on preparing the land for construction work. Over the next year it plans to develop the island extensively,with the aim of making it one of the most exclusive holiday locations in the region.

An offer has just been made to buy the land for £5 million. Paradise Ltd has therefore decided to reappraise the projectin order to decide whether they should still proceed with the project, or should instead accept the offer. If they decideto accept the offer, the sale will take place immediately, incurring legal fees of £20,000. If they reject the offer,development will continue and accommodation will be available for rent in one year’s time.

The company’s project accountant has provided estimates of costs and revenues for the next five years as set outbelow.

1. Total construction costs for the seven hotels on the island are £37 million. Of the total, £2 million has alreadybeen spent in the form of down payments to several construction firms. These down payments are irrecoverable.

2. Total construction costs for the forty luxury self-catering lodges that will be attached to the hotels are £24 million.A down payment of £4 million is required immediately.

3. The cost of furnishing the hotels and lodges is estimated at £3·2 million.

4. Each lodge will have its own private swimming pool. The cost of each pool is expected to be £12,000.

5. Six restaurants will be built on the island at a cost of £15 million. Paradise Ltd has already had to commit to £3 million of these costs in order to attract the chefs it requires. Although these monies have not yet been paidover, Paradise Ltd is contractually bound to pay them, irrespective of whether the project now proceeds.

6. A small parade of shops will be developed at a cost of £4 million.

7. Annual cash overheads are expected to be £2 million for the hotels. Revenues for the hotels are estimated at £13 million per annum.

8. Maintenance costs for each of the lodges will be £7,000 per annum, compared to rental income of £390,000per annum, per lodge.

9. Depreciation totalling £1·5 million per annum will be charged in Paradise Ltd’s accounts for the hotels, lodges,restaurants and shops.

10. The restaurant and shops are expected to generate net income of £4·73 million per annum, in total.

11. Interest on money borrowed to finance the project will be £2·5 million per annum.

All the set-up costs will occur within the next year, before the resort is open. The annual revenues and overheadsrelate to the four years following this. Assume that all cash flows occur at the end of each year, unless otherwisestated, and that there are no terminal values to consider at the end of the four years.

The company’s cost of capital is 10% per annum.

2

Page 96: T10 Question

Required:

(a) Explain the main principles used to differentiate between relevant and irrelevant costs for investmentappraisal, using the information in the question to illustrate your points. (8 marks)

(b) Calculate the project’s net present value (NPV) at the company’s required rate of return. Conclude as towhether the company should accept the offer or continue with the project, giving a reason for yourconclusion. (16 marks)

(c) Calculate the internal rate of return (IRR) for the project, using the discount rates in the tables provided.(4 marks)

(d) State three advantages and three disadvantages of using the IRR as a method of project appraisal.(6 marks)

(e) Briefly outline each of the following stages involved in evaluating capital projects:

(i) Initial investigation of the proposal;

(ii) Detailed evaluation;

(iii) Authorisation;

(iv) Implementation;

(v) Project monitoring;

(vi) Post-completion audit. (6 marks)

(40 marks)

(Workings should be in £’000, to the nearest £’000.)

Present Value Table (extract)Discount Rates (r)

Periods (n) 10% 20%1 0·909 0·8332 0·826 0·6943 0·751 0·5794 0·683 0·4825 0·621 0·402

3 [P.T.O.

Page 97: T10 Question

This is a blank page.Question 2 begins on page 5.

4

Page 98: T10 Question

5 [P.T.O.

2 Chocoholics Ltd sells high quality Belgian chocolates that it buys in ready-made. Its main attraction to customers isthat it gift wraps the items and delivers them to an address of the customer’s choice. The company has just expandedits product range to include flowers.

You are an accounting technician and have been asked to prepare a cash budget for the next six months, incorporatingthe sales of the new products. You have been provided with the following table of estimated revenues and their relativecosts. Other costs are also included in the notes below.

2004 July August September October November December£’000 £’000 £’000 £’000 £’000 £’000

Sales 250 266 282 306 320 330Purchases of chocolates and flowers 125 133 141 153 160 165Administration expenses 155 160 162 165 168 170Packaging costs 112 113 113 113 114 114Miscellaneous expenses 116 116 117 117 118 118Loan repayments 150 – – – 150 –

Notes:1. Opening stocks of chocolates and flowers will amount to £250,000 at 1 July 2004. Closing stocks at

31 December 2004 are estimated at £170,000.2. Suppliers allow one month’s credit. Purchases in June will total £60,000.3. 30% of sales are paid for by cash but the remaining 70% take advantage of the company’s offer of ‘Buy now,

pay in one month’. June 2004 sales are expected to be £140,000.4. Delivery costs are 1% of sales revenue each month, and are paid in the month in which they are incurred.

Administration and miscellaneous expenses are also paid as they are incurred.5. Packaging expenses are paid two months after they are incurred. These costs were £1,000 in May and will be

the same in June.6. The bank charges interest of 1% per month for the overdraft, calculated on the closing bank balance each month,

and payable the following month.7. The bank overdraft at 1 July 2004 is expected to be £155,000.8. The loan repayments above refer to an interest free loan obtained by the company when they moved their

business to a new bank.9. The business has no depreciable fixed assets.

Required:

(a) Prepare a monthly cash budget for EACH of the six months to 31 December 2004, showing the cash balanceat the end of each month. (13 marks)

(b) Prepare a budgeted Profit and Loss account for the six month period ending 31 December 2004.(7 marks)

Note: All workings should be shown clearly in order to score maximum marks. Please show workings in £’000,to the nearest £’000.

(20 marks)

Page 99: T10 Question

6

3 Rant Ltd manufactures and distributes plasma screen televisions to a number of electrical retailers. Due to theincreased popularity of these products, growth has been rapid and the Financial Director (FD) of the company isconcerned that the company is overtrading. Turnover has increased by 250% over the last year, and fixed assets haveincreased by 75%. The bank overdraft limit of £3 million has been exceeded on five occasions in the last year,culminating in a recent threat by the bank to withdraw the facility altogether.

Rant Ltd is a largely family-owned company, with twelve shareholders in total. Whilst the long-term plan involvesmaking a rights issue in two years’ time, none of the current shareholders are in a position to inject new capital intothe company at present. Neither do they wish to issue new shares outside the current group of shareholders, as theydo not want to lose their collective control of the company.

Some preliminary analysis has revealed that debtors are increasing rapidly and raw material stock days have gone upfrom 30 days to 55 days. The FD is concerned that these stock levels are so high. He has asked you, an accountingtechnician, to assist him in reviewing them.

One of the key costs of making a plasma television is the screen. These screens are bought in ready-made at a costof £250 per unit. It costs £150 to place an order for these screens, irrespective of the number of units ordered. Atcurrent sales levels, which are expected to stabilise now, 150,000 screens are needed per annum. The cost of holdingone screen in stock for one year is £15.

Required

(a) Define the terms ‘over-capitalisation’ and ‘overtrading’. (2 marks)

(b) Briefly describe the symptoms of overtrading. Conclude whether Rant Ltd is overtrading, giving reasons foryour conclusion. (6 marks)

(c) Explain the four main types of costs associated with stock management. (4 marks)

(d) Explain how the economic order quantity (EOQ) model can assist in reducing stock costs, AND theassumptions it is based upon. (4 marks)

(e) Calculate the EOQ for screens AND the number of orders to be placed per annum, using the data containedin the question. (4 marks)

(20 marks)

Note: The formula for EOQ is

(where Co represents fixed cost of placing one order;D represents annual demand in terms of units;Ch represents cost of holding one unit of stock per annum.)

2 Co D

Ch

Page 100: T10 Question

7

4 Clean Lens Ltd is a contact lens manufacturer and distributor producing an extensive range of contact lenses that aredistributed direct to customers via the Internet. It is a small company with five shareholders, all of whom are involvedin the running of the company.

The company is in the preliminary stages of developing a new type of contact lens, made of a unique material thatmoulds to the shape of the eye, providing revolutionary comfort for the lens wearer. The company’s projections showthat profits of £1·3 million per annum are expected over the first five years, once sales commence.

In order to proceed with the project, further finance of £1·5 million is required. Clean Lens Ltd expects to fund theproject through a mix of debt and equity finance, and is considering approaching venture capital organisations.

Required:

(a) Briefly explain five factors that Clean Lens Ltd should take into account when deciding on the mix of debtand equity finance. (10 marks)

(b) Identify and discuss five factors that a venture capital organisation should take into account when decidingwhether or not to invest in Clean Lens Ltd. (10 marks)

(20 marks)

End of Question Paper